Docsity
Docsity

Prepare for your exams
Prepare for your exams

Study with the several resources on Docsity


Earn points to download
Earn points to download

Earn points by helping other students or get them with a premium plan


Guidelines and tips
Guidelines and tips

2024 RN Exit Exam Test Bank New Latest Version with All Questions, Exams of Nursing

2024 HES RN Exit Exam Test Bank New Latest Version with All Questions from Actual Past Exam and 100% Correct Answer

Typology: Exams

2023/2024

Available from 07/19/2024

exam-hut
exam-hut 🇺🇸

4.7

(3)

1.2K documents

1 / 88

Toggle sidebar

Related documents


Partial preview of the text

Download 2024 RN Exit Exam Test Bank New Latest Version with All Questions and more Exams Nursing in PDF only on Docsity! 2024 HESI RN Exit Exam Test Bank New Latest Version with All Questions from Actual Past Exam and 100% Correct Answer The nurse enters a 2 year-old child's hospital room in order to administer an oral medication. When the child is asked if he is ready to take his medicine, he immediately says, "No!" What would be the most appropriate next action? A) Leave the room and return five minutes later and give the medicine B) Explain to the child that the medicine must be taken now C) Give the medication to the father and ask him to give it D) Mix the medication with ice cream or applesauce ---------- Correct Answer ----------- A) Leave the room and return five minutes later and give the medicine The nurse is teaching a smoking cessation class and notices there are 2 pregnant women in the group. Which information is a priority for these women? A) Low tar cigarettes are less harmful during pregnancy B) There is a relationship between smoking and low birth weight C) The placenta serves as a barrier to nicotine D) Moderate smoking is effective in weight control ---------- Correct Answer ----------- B) There is a relationship between smoking and low birth weight What is the best way for the nurse to accomplish a health history on a 14 year-old client? A) Have the mother present to verify information B) Allow an opportunity for the teen to express feelings C) Use the same type of language as the adolescent D) Focus the discussion of risk factors in the peer group ---------- Correct Answer --------- -- B) Allow an opportunity for the teen to express feelings Two days prior to discharge from the rehabilitation facility, the nurse is teaching a client who is recovering from Guillain-Barre syndrome about home care. Which actions should the nurse include when providing discharge teaching to the client and spouse? (Select all that apply) A. Review safe transfer strategies B. Develop a nutritional plan C. Help identify community support D. Initiate a rigorous exercise routine E. Provide cooking instructions ----------- Correct Answer ------------ A. Review safe transfer strategies B. Develop a nutritional plan C. Help identify community support A client presents to the emergency department with muscle aches, headache, fever, and describes a recent loss of taste and smell. The nurse obtains a nasal swab for COVID-19 testing. Which action is most important for the nurse to take? A. Place the nasal swab specimen for COVID-19 directly into a biohazard bag B. Move the client to a private room, keep the door closed, and initiate droplet precautions. C. Teach the client to wear a mask, hand wash, and social distance to prevent spreading the virus D. Explain to the client to inform others that they may have been potentially exposed in the last 14 days. ----------- Correct Answer ------------ A. Place the nasal swab specimen for COVID-19 directly into a biohazard bag What principle of HIV disease should the nurse keep in mind when planning care for a newborn who was infected in utero? A) The disease will incubate longer and progress more slowly in this infant B) The infant is very susceptible to infections C) Growth and development patterns will proceed at a normal rate D) Careful monitoring of renal function is indicated ---------- Correct Answer ----------- B) The infant is very susceptible to infections A nurse is doing pre conceptual counseling with a woman who is planning a pregnancy. Which of the following statements suggests that the client understands the connection between alcohol consumption and fetal alcohol syndrome? A) "I understand that a glass of wine with dinner is healthy. "B) "Beer is not really hard alcohol, so I guess I can drink some. "C) "If I drink, my baby may be harmed before I know I am pregnant. " D) "Drinking with meals reduces the effects of alcohol." ---------- Correct Answer --------- -- "C) "If I drink, my baby may be harmed before I know I am pregnant. A client has returned from a cardiac catheterization. Which one of the following assessments would indicate the client is experiencing a complication from the procedure? A) Increased blood pressure B) Increased heart rate C) Loss of pulse in the extremity D) Decreased urine output ---------- Correct Answer ----------- C) Loss of pulse in the extremity A 60 year-old male client had a hernia repair in an outpatient surgery clinic. He is awake and alert, but has not been able to void since he returned from surgery 6 hours ago.He received 1000 mL of IV fluid. Which action would be most likely to help him void? A) Have him drink several glasses of water B) Crede' the bladder from the bottom to the top C) Assist him to stand by the side of the bed to void D) Wait 2 hours and have him try to void again ---------- Correct Answer ----------- C) Assist him to stand by the side of the bed to void D. Palpitations and shortness of breath ---------- Correct Answer ----------- D. Palpitations and shortness of breath A client with a history of heart failure presents to the clinic with a nausea, vomiting, yellow vision and palpitations. Which finding is most important for the nurse to assess to the client? ---------- Correct Answer ----------- Obtain a list of medications taken for cardiac history The pathophysiological mechanism are responsible for ascites related to liver failure? (Select all that apply) ---------- Correct Answer ----------- A. Fluid shifts from intravascular to interstitial area due to decreased serum protein B. Increased hydrostatic pressure in portal circulation increases fluid shifts into abdomen C. Increased circulating aldosterone levels that increase sodium and water retention The nurse is auscultating a client's heart sounds. Which description should the nurse use to document this sound? (Please listen to the audio first to select the option that applies) ---------- Correct Answer ----------- Murmur A client is admitted with a pressure ulcer in the sacral area. The partial thickness wound is 4cm by 7cm, the wound base is red and moist with no exudate and the surrounding skin is intact. Which of the following coverings is most appropriate for this wound? A) Transparent dressing B) Dry sterile dressing with antibiotic ointment C) Wet to dry dressing D) Occlusive moist dressing ---------- Correct Answer ----------- D) Occlusive moist dressing A 30 month-old child is admitted to the hospital unit. Which of the following toys would be appropriate for the nurse to select from the toy room for this child? A) Cartoon stickers B) Large wooden puzzle C) Blunt scissors and paper D) Beach ball ---------- Correct Answer ----------- B) Large wooden puzzle A nurse is to present information about Chinese folk medicine to a group of student nurses. Based on this cultural belief, the nurse would explain that illness is attributed to the A) Yang, the positive force that represents light, warmth, and fullness B) Yin, the negative force that represents darkness, cold, and emptiness C) Use of improper hot foods, herbs and plants D) A failure to keep life in balance with nature and others ---------- Correct Answer -------- --- B) Yin, the negative force that represents darkness, cold, and emptiness A 2 year-old child has just been diagnosed with cystic fibrosis. The child's father asks the nurse "What is our major concern now, and what will we have to deal with in the future?" Which of the following is the best response? A) "There is a probability of life-long complications." B) "Cystic fibrosis results in nutritional concerns that can be dealt with." C) "Thin, tenacious secretions from the lungs are a constant struggle in cystic fibrosis." D) "You will work with a team of experts and also have access to a support group that the family can attend." ---------- Correct Answer ----------- C) "Thin, tenacious secretions from the lungs are a constant struggle in cystic fibrosis." Which type of accidental poisoning would the nurse expect to occur in children under age 6? A) Oral ingestion B) Topical contact C) Inhalation D) Eye splashes ---------- Correct Answer ----------- A) Oral ingestion A client was admitted to the psychiatric unit with a diagnosis of bipolar disorder. He constantly bothers other clients, tries to help the housekeeping staff, demonstrates pressured speech and demands constant attention from the staff. Which activity would be best for the client? A) Reading B) Checkers C) Cards D) Ping-pong ---------- Correct Answer ----------- D) Ping-pong The nurse is caring for a client who has developed cardiac tamponade. Which finding would the nurse anticipate? A) Widening pulse pressure B) Pleural friction rub C) Distended neck veins D) Bradycardia ---------- Correct Answer ----------- C) Distended neck veins Which nursing action is a priority as the plan of care is developed for a 7 year-old child hospitalized for acute glomerulonephritis? A) Assess for generalized edema B) Monitor for increased urinary output C) Encourage rest during hyperactive periods D) Note patterns of increased blood pressure ---------- Correct Answer ----------- D) Note patterns of increased blood pressure The nurse is caring for a child receiving chest physiotherapy (CPT). Which of the following actions by the nurse would be appropriate? A) Schedule the therapy thirty minutes after meals B) Teach the child not to cough during the treatment C) Confine the percussion to the rib cage area D) Place the child in a prone position for the therapy ---------- Correct Answer ----------- C) Confine the percussion to the rib cage area Why is it important for the nurse to monitor blood pressure in clients receiving antipsychotic drugs? A) Orthostatic hypotension is a common side effect B) Most antipsychotic drugs cause elevated blood pressure C) This provides information on the amount of sodium allowed in the diet D) It will indicate the need to institute anti parkinsonian drugs ---------- Correct Answer --- -------- A) Orthostatic hypotension is a common side effect The nurse is teaching the client to select foods rich in potassium to help prevent digitalis toxicity. Which choice indicates the client understands dietary needs? A) Three apricots B) Medium banana C) Naval orange D) Baked potato ---------- Correct Answer ----------- D) Baked potato An 86 year-old nursing home resident who has decreased mental status is hospitalized with pneumonic infiltrates in the right lower lobe. When the nurse assists the client with a clear liquid diet, the client begins to cough. What should the nurse do next? A) Add a thickening agent to the fluids B) Check the client's gag reflex C) Feed the client only solid foods D) Increase the rate of intravenous fluids ---------- Correct Answer ----------- B) Check the client's gag reflex The nurse is planning care for a client with a CVA. Which of the following measures planned by the nurse would be most effective in preventing skin breakdown? A) Place client in the wheelchair for four hours each day B) Pad the bony prominence C) Reposition every two hours D) Massage reddened bony prominence ---------- Correct Answer ----------- C) Reposition every two hours A nurse is assessing several clients in a long term health care facility. Which client is at highest risk for development of decubitus ulcers? A) A 79 year-old malnourished client on bed rest B) An obese client who uses a wheelchair C) A client who had 3 incontinent diarrhea stools D) An 80 year-old ambulatory diabetic client ---------- Correct Answer ----------- C) A client who had 3 incontinent diarrhea stools Constipation is one of the most frequent complaints of elders. When assessing this problem, which action should be the nurse's priority? A) Obtain a complete blood count C) "Since this was the first convulsion, it may not happen again." D) "Long term treatment will prevent future seizures." ---------- Correct Answer ----------- B) "The seizure may or may not mean your child has epilepsy." Alcohol and drug abuse impairs judgment and increases risk taking behavior. What nursing diagnosis best applies? A) Risk for injury B) Risk for knowledge deficit C) Altered thought process D) Disturbance in self-esteem ---------- Correct Answer ----------- A) Risk for injury Which these findings would the nurse more closely associate with anemia in a 10 month-old infant? A) Hemoglobin level of 12 g/dI B) Pale mucosa of the eyelids and lips C) Hypoactivity D) A heart rate between 140 to 160 ---------- Correct Answer ----------- B) Pale mucosa of the eyelids and lips The nurse is caring for a client in hypertensive crisis in an intensive care unit. The priority assessment in the first hour of care is A) Heart rate B) Pedal pulses C) Lung sounds D) Pupil responses ---------- Correct Answer ----------- D) Pupil responses Which of these clients who are all in the terminal stage of cancer is least appropriate to suggest the use of patient controlled analgesia (PCA) with a pump? A) A young adult with a history of Down's syndrome B) A teenager who reads at a 4th grade level C) An elderly client with numerous arthritic nodules on the hands D) A preschooler with intermittent episodes of alertness ---------- Correct Answer ---------- - D) A preschooler with intermittent episodes of alertness The nurse is about to assess a 6 month-old child with nonorganic failure-to thrive (NOFTT). Upon entering the room, the nurse would expect the baby to be A) Irritable and "colicky" with no attempts to pull to standing B) Alert, laughing and playing with a rattle, sitting with support C) Skin color dusky with poor skin turgor over abdomen D) Pale, thin arms and legs, uninterested in surroundings ---------- Correct Answer -------- --- D) Pale, thin arms and legs, uninterested in surroundings As the nurse is speaking with a group of teens which of these side effects of chemotherapy for cancer would the nurse expect this group to be more interested in during the discussion? A) Mouth sores B) Fatigue C) Diarrhea D) Hair loss ---------- Correct Answer ----------- D) Hair loss While caring for a client who was admitted with myocardial infarction (MI) 2 days ago, the nurse notes today's temperature is 101.1 degrees Fahrenheit (38.5 degreesCelsius). The appropriate nursing intervention is to A) Call the health care provider immediately B) Administer acetaminophen as ordered as this is normal at this time C) Send blood, urine and sputum for culture D) Increase the client's fluid intake ---------- Correct Answer ----------- B) Administer acetaminophen as ordered as this is normal at this time A client is admitted for first and second degree burns on the face, neck, anterior chest and hands. The nurse's priority should be A) Cover the areas with dry sterile dressings B) Assess for dyspnea or stridor C) Initiate intravenous therapy D) Administer pain medication ---------- Correct Answer ----------- B) Assess for dyspnea or stridor Which of these clients who call the community health clinic would the nurse ask to come in that day to be seen by the health care provider? A) I started my period and now my urine has turned bright red. B) I am an diabetic and today I have been going to the bathroom every hour. C) I was started on medicine yesterday for a urine infection. Now my lower belly hurts when I go to the bathroom. D) I went to the bathroom and my urine looked very red and it didn't hurt when I went. --- ------- Correct Answer ----------- D) I went to the bathroom and my urine looked very red and it didn't hurt when I went. Which of these parents' comment for a newborn would most likely reveal an initial finding of a suspected pyloric stenosis? A) I noticed a little lump a little above the belly button. B) The baby seems hungry all the time. C) Mild vomiting that progressed to vomiting shooting across the room. D) Irritation and spitting up immediately after feedings. ---------- Correct Answer ----------- C) Mild vomiting that progressed to vomiting shooting across the room. The nurse is assessing a child for clinical manifestations of iron deficiency anemia. Which factor would the nurse recognize as cause for the findings? A) Decreased cardiac output B) Tissue hypoxia C) Cerebral edema D) Reduced oxygen saturation ---------- Correct Answer ----------- B) Tissue hypoxia The nurse would expect the cystic fibrosis client to receive supplemental pancreatic enzymes along with a diet A) High in carbohydrates and proteins B) Low in carbohydrates and proteins C) High in carbohydrates, low in proteins D) Low in carbohydrates, high in proteins ---------- Correct Answer ----------- A) High in carbohydrates and proteins In evaluating the growth of a 12 month-old child, which of these findings would the nurse expect to be present in the infant? A) Increased 10% in height B) 2 deciduous teeth C) Tripled the birth weight D) Head > chest circumference ---------- Correct Answer ----------- C) Tripled the birth weight A Hispanic client in the postpartum period refuses the hospital food because it is "cold." The best initial action by the nurse is to A) 1Have the unlicensed assistive personnel (UAP) reheat the food if the client wishes B) Ask the client what foods are acceptable or bad C) Encourage her to eat for healing and strength D) Schedule the dietitian to meet with the client as soon as possible ---------- Correct Answer ----------- B) Ask the client what foods are acceptable or bad The father of an 8 month-old infant asks the nurse if his infant's vocalizations are normal for his age. Which of the following would the nurse expect at this age? A) Cooing B) Imitation of sounds C) Throaty sounds D) Laughter ---------- Correct Answer ----------- B) Imitation of sounds The nurse should recognize that physical dependence is accompanied by what findings when alcohol consumption is first reduced or ended? A) Seizures B) Withdrawal C) Craving D) Marked tolerance ---------- Correct Answer ----------- B) Withdrawal Immediately following an acute battering incident in a violent relationship, the batterer may respond to the partner's injuries by A) Seeking medical help for the victim's injuries B) Minimizing the episode and underestimating the victim's injuries C) Contacting a close friend and asking for help D) Being very remorseful and assisting the victim with medical care ---------- Correct Answer ----------- B) Minimizing the episode and underestimating the victim's injuries D) Sedimentation Rate ---------- Correct Answer ----------- C) Bilirubin The nurse is discussing nutritional requirements with the parents of an 18 month-old child. Which of these statements about milk consumption is correct? A) May drink as much milk as desired B) Can have milk mixed with other foods C) Will benefit from fat-free cow's milk D) Should be limited to 3-4 cups of milk daily ---------- Correct Answer ----------- D) Should be limited to 3-4 cups of milk daily The nurse is talking with a client. The client abruptly says to the nurse, "The moon is full. Astronauts walk on the moon. Walking is a good health habit." The client's behavior most likely indicates A) Neologisms B) Dissociation C) Flight of ideas D) Word salad ---------- Correct Answer ----------- C) Flight of ideas A mother asks about expected motor skills for a 3 year-old child. Which of the following would the nurse emphasize as normal at this age? A) Jumping rope B) Tying shoelaces C) Riding a tricycle D) Playing hopscotch ---------- Correct Answer ----------- C) Riding a tricycle A home health nurse is caring for a client with a pressure sore that is red, with serous drainage, is 2 inches in diameter with loss of subcutaneous tissue. The appropriate dressing for this wound is A) A transparent film dressing B) Wet dressing with debridement granules C) Wet to dry with hydrogen peroxide D) Moist saline dressing ---------- Correct Answer ----------- D) Moist saline dressing Which bed position is preferred for use with a client in an extended care facility on falls risk prevention protocol? A) All 4 side rails up, wheels locked, bed closest to door B) Lower side rails up, bed facing doorway C) Knees bent, head slightly elevated, bed in lowest position D) Bed in lowest position, wheels locked, place bed against wall ---------- Correct Answer ----------- D) Bed in lowest position, wheels locked, place bed against wall The nurse is talking to parents about nutrition in school aged children. Which of the following is the most common nutritional disorder in this age group? A) Bulimia B) Anorexia C) Obesity D) Malnutrition ---------- Correct Answer ----------- C) Obesity At the geriatric day care program a client is crying and repeating "I want to go home. Call my daddy to come for me." The nurse should A) Invite the client to join the exercise group B) Tell the client you will call someone to come for her C) Give the client simple information about what she will be doing D) Firmly direct the client to her assigned group activity ---------- Correct Answer ---------- - C) Give the client simple information about what she will be doing A victim of domestic violence states to the nurse, "If only I could change and be how my companion wants me to be, I know things would be different." Which would be the best response by the nurse? A) "The violence is temporarily caused by unusual circumstances, don't stop hoping for a change. "B) "Perhaps, if you understood the need to abuse, you could stop the violence. "C) "No one deserves to be beaten. Are you doing anything to provoke your spouse into beating you?" D) "Batterers lose self-control because of their own internal reasons, not because of what their partner did or did not do." ---------- Correct Answer ----------- D) "Batterers lose self- control because of their own internal reasons, not because of what their partner did or did not do." A 38 year-old female client is admitted to the hospital with an acute exacerbation of asthma. This is her third admission for asthma in 7 months. She describes how she doesn't really like having to use her medications all the time. Which explanation by the nurse best describes the long-term consequence of uncontrolled airway inflammation? A) Degeneration of the alveoli B) Chronic broncho constriction of the large airways C) Lung remodeling and permanent changes in lung function D) Frequent pneumonia ---------- Correct Answer ----------- C) Lung remodeling and permanent changes in lung function A mother wants to switch her 9 month-old infant from an iron fortified formula to whole milk because of the expense. Upon further assessment, the nurse finds that the baby eats table foods well, but drinks less milk than before. What is the best advice by the nurse? A) Change the baby to whole milk B) Add chocolate syrup to the bottle C) Continue with the present formula D) Offer fruit juice frequently ---------- Correct Answer ----------- C) Continue with the present formula Privacy and confidentiality of all client information is legally protected. In which of these situations would the nurse make an exception to this practice? A) When a family member offers information about their loved one B) When the client threatens self-harm and harm to others C) When the health care provider decides the family has a right to know the client's diagnosis D) When a visitor insists that the visitor has been given permission by the client ---------- Correct Answer ----------- B) When the client threatens self-harm and harm to others The nurse is caring for a client who is in the late stage of multiple myeloma. Which of the following should be included in the plan of care? A) Monitor for hyperkalemia B) Place in protective isolation C) Precautions with position changes D) Administer diuretics as ordered ---------- Correct Answer ----------- C) Precautions with position changes The nurse is making a home visit to a client with chronic obstructive pulmonary disease (COPD). The client tells the nurse that he used to be able to walk from the house to the mailbox without difficulty. Now, he has to pause to catch his breath halfway through the trip. Which diagnosis would be most appropriate for this client based on this assessment? A) Activity intolerance caused by fatigue related to chronic tissue hypoxia B) Impaired mobility related to chronic obstructive pulmonary disease C) Self-care deficit caused by fatigue related to dyspnea D) Ineffective airway clearance related to increased bronchial secretions ---------- Correct Answer ----------- A) Activity intolerance caused by fatigue related to chronic tissue hypoxia The nurse admits a client newly diagnosed with hypertension. What is the best method for assessing the blood pressure? A) Standing and sitting B) In both arms C) After exercising D) Supine position ---------- Correct Answer ----------- B) In both arms The nurse is caring for residents in a long term care setting for the elderly. Which of the following activities will be most effective in meeting the growth and development needs for persons in this age group? A) Aerobic exercise classes B) Transportation for shopping trips C) Reminiscence groups D) Regularly scheduled social activities ---------- Correct Answer ----------- C) Reminiscence groups Post-procedure nursing interventions for electroconvulsive therapy include A) Applying hard restraints if seizure occurs B) Expecting client to sleep for 4 to 6 hours C) Remaining with client until oriented D) Altered patterns of urinary elimination related to nocturia ---------- Correct Answer ----- ------ D) Altered patterns of urinary elimination related to nocturia A newly admitted adult client has a diagnosis of hepatitis A. The charge nurse should reinforce to the staff members that the most significant routine infection control strategy, in addition to hand washing, to be implemented is which of these? A) Apply appropriate signs outside and inside the room B) Apply a mask with a shield if there is a risk of fluid splash C) Wear a gown to change soiled linens from incontinence D) Have gloves on while handling bedpans with feces ---------- Correct Answer ----------- D) Have gloves on while handling bedpans with feces Which of these clients with associated lab reports is a priority for the nurse to report to the public health department within the next 24 hours? A) An infant with a positive culture of stool for Shigella B) An elderly factory worker with a lab report that is positive for acid-fast bacillus smear C) A young adult commercial pilot with a positive histopathological examination from an induced sputum for Pneumocystis carinii D) A middle-aged nurse with a history of varicella-zoster virus and with crops of vesicles on an erythematous base that appear on the skin ---------- Correct Answer ----------- B) An elderly factory worker with a lab report that is positive for acid-fast bacillus smear A client is diagnosed with methicillin resistant staphylococcus aureus pneumonia. What type of isolation is most appropriate for this client? A) Reverse B) Airborne C) Standard precautions D) Contact ---------- Correct Answer ----------- D) Contact The school nurse is teaching the faculty the most effective methods to prevent the spread of lice in the school. The information that would be most important to include would be which of these statements? A) "The treatment requires reapplication in 8 to 10 days." B) "Bedding and clothing can be boiled or steamed." C) Children are not to share hats, scarves and combs. D) Nit combs are necessary to comb out nits. ---------- Correct Answer ----------- C) Children are not to share hats, scarves and combs. During the care of a client with a salmonella infection, the primary nursing intervention to limit transmission is which of these approaches? A) Wash hands thoroughly before and after client contact B) Wear gloves when in contact with body secretions C) Double glove when in contact with feces or vomitus D) Wear gloves when disposing of contaminated linens ---------- Correct Answer ---------- - A) Wash hands thoroughly before and after client contact A nurse is reinforcing teaching with a client about compromised host precautions. The client is receiving filgrastim (Neupogen) for neutropenia. The selection of which lunch suggests the client has learned about necessary dietary changes? A) Grilled chicken sandwich and skim milk B) Roast beef, mashed potatoes, and green beans C) Peanut butter sandwich, banana, and iced tea D) Barbecue beef, baked beans, and cole slaw ---------- Correct Answer ----------- B) Roast beef, mashed potatoes, and green beans After talking with her partner, a client voluntarily admitted herself to the substance abuse unit. After the second day on the unit the client states to the nurse, "My husband told me to get treatment or he would divorce me. I don't believe I really need treatment but I don't want my husband to leave me." Which response by the nurse would assist the client? A) "In early recovery, it's quite common to have mixed feelings, but unmotivated people can't get well." B) "In early recovery, it's quite common to have mixed feelings, but I didn't know you had been pressured to come." C) "In early recovery it's quite common to have mixed feelings, perhaps it would be best to seek treatment on an out client bases. " D) " In early recovery, it's quite common to have mixed feelings. Let's discuss the benefits of sobriety for you." ---------- Correct Answer ----------- D) " In early recovery, it's quite common to have mixed feelings. Let's discuss the benefits of sobriety for you." A neonate born 12 hours ago to a methadone maintained woman is exhibiting a hyperactive MORO reflex and slight tremors. The newborn passes loose, watery stool. Which of these is a nursing priority? A) Hold the infant at frequent intervals. B) Assess for neonatal withdrawal syndrome C) Offer fluids to prevent dehydration D) Administer paregoric to stop diarrhea ---------- Correct Answer ----------- B) Assess for neonatal withdrawal syndrome The nurse is caring for a post myocardial infarction client in an intensive care unit. It is noted that urinary output has dropped from 60 -70 ml per hour to 30 ml per hour. This change is most likely due to A) Dehydration B) Diminished blood volume C) Decreased cardiac output D) Renal failure ---------- Correct Answer ----------- C) Decreased cardiac output The primary nursing diagnosis for a client with congestive heart failure with pulmonary edema is A) Pain B) Impaired gas exchange C) Cardiac output altered: decreased D) Fluid volume excess ---------- Correct Answer ----------- C) Cardiac output altered: decreased The nurse is performing a developmental assessment on an 8 month-old. Which finding should be reported to the health care provider? A) Lifts head from the prone position B) Rolls from abdomen to back C) Responds to parents' voices D) Falls forward when sitting ---------- Correct Answer ----------- D) Falls forward when sitting A client has received her first dose of fluphenazine (Prolixin) 2 hours ago. She suddenly experiences torticollis and involuntary spastic muscle movement. In addition to administering the ordered anticholinergic drug, what other measure should the nurse implement? A) Have respiratory support equipment available B) Immediately place her in the seclusion room C) Assess the client for anxiety and agitation D) Administer PRN dose of IM antipsychotic medication ---------- Correct Answer ---------- - A) Have respiratory support equipment available The nurse walks into a client's room and finds the client lying still and silent on the floor. The nurse should first A) Assess the client's airway B) Call for help C) Establish that the client is unresponsive D) See if anyone saw the client fall ---------- Correct Answer ----------- C) Establish that the client is unresponsive The nurse is caring for a client 2 hours after a right lower lobectomy. During the evaluation of the water-seal chest drainage system, it is noted that the fluid level bubbles constantly in the water seal chamber. On inspection of the chest dressing and tubing, the nurse does not find any air leaks in the system. The next best action for the nurse is to A) Check for subcutaneous emphysema in the upper torso B) Reposition the client to a position of comfort C) Call the health care provider as soon as possible D) Check for any increase in the amount of thoracic drainage ---------- Correct Answer -- --------- A) Check for subcutaneous emphysema in the upper torso The nurse is teaching a client with dysrhythmia about the electrical pathway of an impulse as it travels through the heart. Which of these demonstrates the normal pathway? A) AV node, SA node, Bundle of His, Purkinje fibers B) Purkinje fibers, SA node, AV node, Bundle of His C) Bundle of His, Purkinje fibers, SA node , AV node A client tells the nurse, "I have something very important to tell you if you promise not to tell." The best response by the nurse is A) "I must document and report any information." B) "I can't make such a promise." C) "That depends on what you tell me." D) "I must report everything to the treatment team." ---------- Correct Answer ----------- C) Was minimally responsive to voice and touch Which task could be safely delegated by the nurse to an unlicensed assistive personnel (UAP)? A) Be with a client who self-administers insulin B) Cleanse and dress a small decubitus ulcer C) Monitor a client's response to passive range of motion exercises D) Apply and care for a client's rectal pouch ---------- Correct Answer ----------- D) Apply and care for a client's rectal pouch A client asks the nurse to call the police and states: "I need to report that I am being abused by a nurse." The nurse should first A) Focus on reality orientation to place and person B) Assist with the report of the client's complaint to the police C) Obtain more details of the client's claim of abuse D) Document the statement on the client's chart with a report to the manager ---------- Correct Answer ----------- C) Obtain more details of the client's claim of abuse A nurse from the maternity unit is floated to the critical care unit because of staff shortage on the evening shift. Which client would be appropriate to assign to this nurse? A client with A) A Dopamine drip IV with vital signs monitored every 5 minutes B) A myocardial infarction that is free from pain and dysrhythmias C) A tracheotomy of 24 hours in some respiratory distress D) A pacemaker inserted this morning with intermittent capture ---------- Correct Answer ----------- B) A myocardial infarction that is free from pain and dysrhythmias An unlicensed assistive personnel (UAP), who usually works on a surgical unit is assigned to float to a pediatric unit. Which question by the charge nurse would be most appropriate when making delegation decisions? A) "How long have you been a UAP and what units you have worked on?" B) "What type of care do you give on the surgical unit and what ages of clients?" C) "What is your comfort level in caring for children and at what ages?" D) "Have you reviewed the list of expected skills you might need on this unit?" ---------- Correct Answer ----------- D) "Have you reviewed the list of expected skills you might need on this unit?" A client frequently admitted to the locked psychiatric unit repeatedly compliments and invites one of the nurses to go out on a date. The nurse's response should be to A) Ask to not be assigned to this client or to work on another unit B) Tell the client that such behavior is inappropriate C) Inform the client that hospital policy prohibits staff to date clients D) Discuss the boundaries of the therapeutic relationship with the client ---------- Correct Answer ----------- D) Discuss the boundaries of the therapeutic relationship with the client In order to enhance a client's response to medication for chest pain from acute angina, the nurse should emphasize A) Learning relaxation techniques B) Limiting alcohol use C) Eating smaller meals D) Avoiding passive smoke ---------- Correct Answer ----------- A) Learning relaxation techniques The nurse is caring for 2 children who have had surgical repair of congenital heart defects. For which defect is it a priority to assess for findings of heart conduction disturbance? A) Arterial septal defect B) Patent ductus arteriosus C) Aortic stenosis D) Ventricular septal defect ---------- Correct Answer ----------- D) Ventricular septal defect Clients with mitral stenosis would likely manifest findings associated with congestion in the A) Pulmonary circulation B) Descending aorta C) Superior vena cava D) Bundle of His ---------- Correct Answer ----------- A) Pulmonary circulation While planning care for a preschool aged child, the nurse understands needs. Which of the following would be of the most concern to the nurse? A) Playing imaginatively B) Expressing shame C) Identifying with family D) Exploring the playroom ---------- Correct Answer ----------- B) Expressing shame A client has been receiving lithium (Lithane) for the past two weeks for the treatment of bipolar illness. When planning client teaching, what is most important to emphasize to the client? A) Maintain a low sodium diet B) Take a diuretic with lithium C) Come in for evaluation of serum lithium levels every 1-3 months D) Have blood lithium levels drawn during the summer months ---------- Correct Answer ----------- D) Have blood lithium levels drawn during the summer months While teaching a client about their medications, the client asks how long it will take before the effects of lithium take place. What is the best response of the nurse? A) Immediately B) Several days C) 2 weeks D) 1 month ---------- Correct Answer ----------- C) 2 weeks The nursing intervention that best describes treatment to deal with the behaviors of clients with personality disorders include A) Pointing out inconsistencies in speech patterns to correct thought disorders B) Accepting client and the client's behavior unconditionally C) Encouraging dependency in order to develop ego controls D) Consistent limit-setting enforced 24 hours per day ---------- Correct Answer ----------- D) Consistent limit-setting enforced 24 hours per day Following a cocaine high, the user commonly experiences an extremely unpleasant feeling called A) Craving B) Crashing C) Outward bound D) Nodding out ---------- Correct Answer ----------- B) Crashing The nurse asks a client with a history of alcoholism about the client's drinking behavior. The client states "I didn't hurt anyone. I just like to have a good time, and drinking helps me to relax." The client is using which defense mechanism? A) Denial B) Projection C) Intellectualization D) Rationalization ---------- Correct Answer ----------- D) Rationalization One reason that domestic violence remains extensively undetected is A) Few battered victims seek medical care B) There is typically a series of minor, vague complaints C) Expenses due to police and court costs are prohibitive D) Very little knowledge is currently known about batterers and battering relationships -- -------- Correct Answer ----------- B) There is typically a series of minor, vague complaints A client develops volume overload from an IV that has infused too rapidly. What assessment would the nurse expect to find? A) S3 heart sound B) Thready pulse C) Flattened neck veins D) Hypoventilation ---------- Correct Answer ----------- A) S3 heart sound client information in a uniform pocket. Which action should the nurse take? ----------- Correct Answer ----------- a. File a detailed incident report with the specific hiring facility. 7. The nurse is evaluating a tertiary prevention program for clients with cardiovascular disease implemented in a rural health clinic. Which outcome indicate the program is effective? ----------- Correct Answer ----------- c. Clients who incurred disease complications promptly received rehabilitation. 8. The nurse is caring for a client with chronic obstructive pulmonary disease (COPD) who uses oxygen at 2 L/minute per nasal cannula continuously. The nurse observes that the client is having increased shortness of breath with respirations at 23 breaths/minute. Which action should the nurse implement first? ----------- Correct Answer ----------- d. Assess the delivery mechanism of the oxygen tank, tubing, and cannula. 9. Which statement by a client who is 24 hours post-subtotal thyroidectomy requires an immediate investigation by the nurse? ----------- Correct Answer ----------- When I get out of bed quickly, I feel a little dizzy." 10. An older adult male who is in his early 70's is admitted to the emergency department because of a COPD exacerbation. This client is struggling to breathe and the healthcare team is preparing for endotracheal intubation. The spouse's wife, who is 30 years younger than the client, asks the nurse to stop the procedure and provide the nurse a copy of the client's living will. Which action should the nurse take? ----------- Correct Answer ----------- b. Notify the healthcare provider of the client's wishes. 11. An unlicensed assistive personnel (UAP) is assigned to provide personal care for a client whose prescribed activity is bedrest with bedside commode use. The UAP reports to the nurse that the client is so obese that the UAP feels unable to safely assist the client in transferring from the bed to the bedside commode. How should the nurse respond? ----------- Correct Answer ----------- c. Advice the client to maintain bedrest so that safety can be ensured. 12. A nurse determines that more than 25% of the students at a middle school are overweight. The nurse presents the information at the parent-teacher meeting. What action is most important for the nurse to include in the meeting? ----------- Correct Answer ----------- c. Distribute a shopping list of suggested healthy snack items. 13. After several months of chronic fatigue, morning stiffness, and join pain, a young adult is diagnosed with rheumatoid arthritis, and the healthcare provider prescribes prednisone. Which education should the nurse provide the client with regard to taking prednisone? ----------- Correct Answer ----------- c. If sequential doses are missed, notify the healthcare provider. 14. The psychiatric nurse is caring for clients on an adolescent unit. Which client requires the nurse's immediate attention? ----------- Correct Answer ----------- c. An 18- year-old client with antisocial behavior who is being yelled at by other clients 15. The nurse caring for a child with mononucleosis can expect the child to exhibit which symptoms? ----------- Correct Answer ----------- b. Ear pain and fever. 16. A client arrives for an annual physical exam and complains of having calf pain. The client's health history reveals peripheral atrial disease. Which question should the nurse ask the client about expected finding related to chronic arterial symptoms? ----------- Correct Answer ----------- b. Does the calf pain occur when walking short distances? 17. The nurse is preparing to send a client to the cardiac catheterization lab for an angioplasty. Which client report is most important for them to explore further prior to the start of the procedure? ----------- Correct Answer ----------- d. Experience facial swelling after eating crab. 18. The nurse is assessing a 4-year-old child with eczema. The child's skin is dry and scaly, and the mother reports that the child frequently scratches the lesions on the skin to the point of causing bleeding. Which guideline is indicated for care of this child? ------- ---- Correct Answer ----------- b. Apply baby lotion to the skin twice daily. 19. A new mother on the postpartum unit runs out of the room screaming that her newborn infant's crib is empty and the baby is missing. What action should the nurse take first? ----------- Correct Answer ----------- d. Match ID bands of all infants and mothers on the unit. 20. While providing a health history, a female client tells the clinic nurse that she frequently thinks about hurting herself. Which question is most important for the nurse to ask? ----------- Correct Answer ----------- c. "Have you thought about taking your life?" 21. A college student brings a dorm roommate to the campus clinic because the roommate has been talking to someone who is not present. The client tells the nurse that the voices are saying, "kill, kill." What question should the nurse ask the client next? ----------- Correct Answer ----------- c. "Are you planning to obey the voices?" 22. The nurse is developing a plan of care for a client who reports tingling of the feet and who is newly diagnosed with peripheral vascular disease. Which outcome should the nurse include in the plan of care for this client? ----------- Correct Answer ----------- d. The client's skin on the lower legs will be intact at the next clinical visit. 23. When conducting diet teaching for a client who was diagnosed with hypertension, which food should the nurse encourage the client to eat? (select all that apply.) ----------- Correct Answer ----------- a. . Fruits without sauce c. Fresh or frozen vegetables without sauce. 24. A client with bacterial meningitis is receiving phenytoin. Which assessment finding indication to the nurse that the client is experiencing a therapeutic response to the phenytoin? ----------- Correct Answer ----------- c. Absence of seizure activity for the duration of treatment. 25. The nurse observes a client prepare a meal in the kitchen of a rehabilitation facility prior to discharge. Which behaviors indicate the client understands how to maintain balance safely? (Select all that apply) ----------- Correct Answer ----------- a. Brings a heavy can close to body before lifting. b. Locks knees while preparing food on the counter. 26. An older client is admitted to the hospital because of recurring transient ischemic attacks. Neurological serial assessments for the past 24 hours were within normal limits. One day after admission, the client suddenly becomes confused and combative indicating impaired mental status (IMS). What intervention should the nurse implement first? ----------- Correct Answer ----------- b. Reduce environmental stimuli. 27. The charge nurse in an extended care facility is organizing unit activities for the day. Which action may be safely delegated to the practical nurse (PN)? ----------- Correct Answer ----------- Establish blood pressure parameters for client monitoring 28. During shift report, the charge nurse receives notice of several problems. Which problem should the nurse address first? ----------- Correct Answer ----------- d. A bucket of water was spilled in the hallway. 29. An older adult client with chronic emphysema is admitted to the emergency room from home with acute onset of weakness, palpitations, and vomiting. Which information is most important for the nurse to obtain during the initial interview? ----------- Correct Answer ----------- Recent compliance with prescribed medications 30. The nurse is feeding an older adult who was admitted with aspiration pneumonia. The client is weak and begins coughing while attempting to drink through a straw. Which intervention should the nurse implement? ----------- Correct Answer ----------- b. Monitor the client when using a straw for liquids. 31. A male client with right-sided weakness calls for assistance with ambulating to the bathroom. What action should the nurse implement? ----------- Correct Answer ----------- b. Stand on the client's right side as he walks. 32. An older client is admitted to the psychiatric unit for assessment of a recent onset of dementia. The nurse notes that in the evening this client often becomes restless, confused, and agitated. Which intervention is most important for the nurse to implement? ----------- Correct Answer ----------- b. Ensure that the client is assigned to a room close to the nurses' station. reported by the nurse immediately to the healthcare provider? ----------- Correct Answer ----------- B) Fever of 103 degrees Fahrenheit (39.5 degrees Celsius) 53. A client who had a vasectomy is in the post recovery unit at an outpatient clinic. Which of these points is most important to be reinforced by the nurse? ----------- Correct Answer ----------- A) Until the health care provider has determined that your ejaculate doesn't contain sperm, continue to use another form of contraception. 54. A client who is to have antineoplastic chemotherapy tells the nurses of a fear of being sick all the time and wishes to try acupuncture. Which of these beliefs stated by the client would be incorrect about acupuncture? ----------- Correct Answer ----------- C) The flow of life is believed to flow through major pathways or nerve clusters in your body. 55. The nurse is discussing with a group of students the disease Kawasaki. What statement made by a student about Kawasaki disease is incorrect? ----------- Correct Answer ----------- C) Kawasaki disease occurs most often in boys, children younger than age 5 and children of Hispanic descent 56. A client has viral pneumonia affecting 2/3 of the right lung. What would be the best position to teach the client to lie in every other hour during first 12 hours after admission? ----------- Correct Answer ----------- A) Side-lying on the left with the head elevated 10 degrees 57. A client has an indwelling catheter with continuous bladder irrigation after undergoing a transurethral resection of the prostate (TURP) 12 hours ago. Which finding at this time should be reported to the health care provider? ----------- Correct Answer ----------- minimal drainage into the urinary collection bag 58. A nurse is performing CPR on an adult who went into cardiopulmonary arrest. Another nurse enters the room in response to the call. After checking the client's pulse and respirations, what should be the function of the second nurse? ----------- Correct Answer ----------- C) Participate with the compressions or breathing 59. The nurse assesses a 72 year-old client who was admitted for right sided congestive heart failure. Which of the following would the nurse anticipate finding? ------- ---- Correct Answer ----------- B) Jugular vein distention 60. A client with heart failure has a prescription for digoxin. The nurse is aware that sufficient potassium should be included in the diet because hypokalemia in combination with this medication ----------- Correct Answer ----------- Can predispose to dysrhythmias 61. A nurse assesses a young adult in the emergency room following a motor vehicle accident. Which of the following neurological signs is of most concern? ----------- Correct Answer ----------- Pupils fixed and dilated 62. A 14 year-old with a history of sickle cell disease is admitted to the hospital with a diagnosis of vaso-occlusive crisis. Which statements by the client would be most indicative of the etiology of this crisis? ----------- Correct Answer ----------- D)"I went to the health care provider last week for a cold and I have gotten worse." 63. Which these findings would the nurse more closely associate with anemia in a 10 month-old infant? ----------- Correct Answer ----------- Pale mucosa of the eyelids and lips 64. The nurse is caring for a client in hypertensive crisis in an intensive care unit. The priority assessment in the first hour of care is ----------- Correct Answer ----------- Pupil responses 65. Which of these clients who are all in the terminal stage of cancer is least appropriate to suggest the use of patient controlled analgesia (PCA) with a pump? ----------- Correct Answer ----------- D) A preschooler with intermittent episodes of alertness 66. The nurse is about to assess a 6 month-old child with nonorganic failure-to thrive (NOFTT). Upon entering the room, the nurse would expect the baby to be ----------- Correct Answer ----------- D) Pale, thin arms and legs, uninterested in surroundings 67. As the nurse is speaking with a group of teens which of these side effects of chemotherapy for cancer would the nurse expect this group to be more interested in during the discussion? ----------- Correct Answer ----------- D) Hair loss 68. While caring for a client who was admitted with myocardial infarction (MI) 2 days ago, the nurse notes today's temperature is 101.1 degrees Fahrenheit (38.5 degrees Celsius). The appropriate nursing intervention is to ----------- Correct Answer ----------- Administer acetaminophen as ordered as this is normal at this time 69. A client is admitted for first and second degree burns on the face, neck, anterior chest and hands. The nurse's priority should be ----------- Correct Answer ----------- B) Assess for dyspnea or stridor 70. Which of these clients who call the community health clinic would the nurse ask to come in that day to be seen by the health care provider? ----------- Correct Answer ------- ---- I went to the bathroom and my urine looked very red and it didn't hurt when I went. 71. A middle aged woman talks to the nurse in the health care provider's office about uterine fibroids also called leiomyomas or myomas. What statement by the woman indicates more education is needed? ----------- Correct Answer ----------- Fibroids that cause no problems still need to be taken out. 72. An elderly client admitted after a fall begins to seize and loses consciousness. What action by the nurse is appropriate to do next? ----------- Correct Answer ----------- A) Stay with client and observe for airway obstruction 73. A nurse is providing care to a primigravida whose membranes spontaneously ruptured (ROM) 4 hours ago. Labor is to be induced. At the time of the ROM the vital signs were T-99.8 degrees F, P-84, R-20, BP-130/78, and fetal heart tones (FHT) 148 beats/min. Which assessment findings taken now may be an early indication that the client is developing a complication of labor? ----------- Correct Answer ----------- A) FHT 168 beats/min 74. A client with pneumococcal pneumonia had been started on antibiotics 16 hours ago. During the nurse's initial evening rounds the nurse notices a foul smell in the room. The client makes all of these statements during their conversation. Which statement would alert the nurse to a complication? ----------- Correct Answer ----------- B) "I have been coughing up foul-tasting, brown, thick sputum." 75. The nurse is performing an assessment on a client in congestive heart failure. Auscultation of the heart is most likely to reveal ----------- Correct Answer ----------- S3 ventricular gallop 76. Which of these observations made by the nurse during an excretory urogram indicate a complicaton? ----------- Correct Answer ----------- B) The client's entire body turns a bright red color 77. A client is diagnosed with a spontaneous pneumothorax necessitating the insertion of a chest tube. What is the best explanation for the nurse to provide this client? ---------- - Correct Answer ----------- "The tube will remove excess air from your chest." 78. The nurse is reviewing laboratory results on a client with acute renal failure. Which one of the following should be reported immediately? ----------- Correct Answer ----------- Serum potassium 6 mEq/L 79. The nurse is caring for a client undergoing the placement of a central venous catheter line. Which of the following would require the nurse's immediate attention? ------ ----- Correct Answer ----------- C) Dyspnea 80. The nurse is performing a physical assessment on a client who just had an endotracheal tube inserted. Which finding would call for immediate action by the nurse? ----------- Correct Answer ----------- C) Pulse oximetry of 88 81. A nurse checks a client who is on a volume-cycled ventilator. Which finding indicates that the client may need suctioning? ----------- Correct Answer ----------- D) restlessness 101. A client is recovering from a hip replacement and is taking Tylenol #3 every 3 hours for pain. In checking the client, which finding suggests a side effect of the analgesic? ----------- Correct Answer ----------- D) No bowel movement for 3 days Skip 102. A client is being maintained on heparin therapy for deep vein thrombosis. The nurse must closely monitor which of the following laboratory values? ----------- Correct Answer ----------- C) Activated PTT 103. A client with amyotrophic lateral sclerosis has a percutaneous endoscopic gastrostomy (PEG) tube for the administration of feedings and medications. Which nursing action is appropriate? ----------- Correct Answer ----------- D) Flush adequately with water before and after using the tube 104. The nurse has given discharge instructions to parents of a child on phenytoin (Dilantin). Which of the following statements suggests that the teaching was effective? -- --------- Correct Answer ----------- B) "Our child should brush and floss carefully after every meal." 105. The nurse is caring for a client with clinical depression who is receiving a MAO inhibitor. When providing instructions about precautions with this medication, which action should the nurse stress to the client as important? ----------- Correct Answer ------- ---- Avoid chocolate and cheese 106. A parent asks the school nurse how to eliminate lice from their child. What is the most appropriate response by the nurse? ----------- Correct Answer ----------- D) Application of pediculicides 107. A client diagnosed with cirrhosis of the liver and ascites is receiving Spironolactone (Aldactone). The nurse understands that this medication spares elimination of which element? ----------- Correct Answer ----------- B) Potassium 108. The nurse is caring for a client receiving a blood transfusion who develops urticaria one-half hour after the transfusion has begun. What is the first action the nurse should take? ----------- Correct Answer ----------- A) Stop the infusion 109. Discharge instructions for a client taking alprazolam (Xanax) should include which of the following? ----------- Correct Answer ----------- B) Sudden cessation of alprazolam (Xanax) can cause rebound insomnia and nightmares 110. A client has received 2 units of whole blood today following an episode of GI bleeding. Which of the following laboratory reports would the nurse monitor most closely? ----------- Correct Answer ----------- Hemoglobin and hematocrit 111. A client is receiving intravenous heparin therapy. What medication should the nurse have available in the event of an overdose of heparin? ----------- Correct Answer -- --------- Protamine 112. The nurse has been teaching a client with Insulin Dependent Diabetes Mellitus. Which statement by the client indicates a need for further teaching? ----------- Correct Answer ----------- D) "I always make sure to shake the NPH bottle hard to mix it well." 113. Why is it important for the nurse to monitor blood pressure in clients receiving antipsychotic drugs? ----------- Correct Answer ----------- Orthostatic hypotension is a common side effect 114. The nurse is teaching the client to select foods rich in potassium to help prevent digitalis toxicity. Which choice indicates the client understands dietary needs? ----------- Correct Answer ----------- D) Baked potato 115. An 86 year-old nursing home resident who has decreased mental status is hospitalized with pneumonic infiltrates in the right lower lobe. When the nurse assists the client with a clear liquid diet, the client begins to cough. What should the nurse do next? ----------- Correct Answer ----------- B) Check the client's gag reflex 116. The nurse is planning care for a client with a CVA. Which of the following measures planned by the nurse would be most effective in preventing skin breakdown? ----------- Correct Answer ----------- C) Reposition every two hours 117. A nurse is assessing several clients in a long term health care facility. Which client is at highest risk for development of decubitus ulcers? ----------- Correct Answer ----------- A 79 year-old malnourished client on bed rest 118. After a client has an enteral feeding tube inserted, the most accurate method for verification of placement is ----------- Correct Answer ----------- Abdominal x-ray 119. The nurse is instructing a 65 year-old female client diagnosed with osteoporosis. The most important instruction regarding exercise would be to ----------- Correct Answer ----------- Exercise doing weight bearing activities 120. Which bed position is preferred for use with a client in an extended care facility on falls risk prevention protocol? ----------- Correct Answer ----------- D) Bed in lowest position, wheels locked, place bed against wall 121. When administering enteral feeding to a client via a jejunostomy tube, the nurse should administer the formula ----------- Correct Answer ----------- B) Continuously 2. The nurse is teaching an 87 year-old client methods for maintaining regular bowel movements. The nurse would caution the client to AVOID ----------- Correct Answer ------ ----- C) Laxatives 123. A nurse is providing care to a 63 year-old client with pneumonia. Which intervention promotes the client's comfort? ----------- Correct Answer ----------- C) Keep conversations short 124. The nurse is caring for a 7 year-old with acute glomerulonephritis (AGN). Findings include moderate edema and oliguria. Serum blood urea nitrogen and creatinine are elevated. What dietary modifications are most appropriate? ----------- Correct Answer ---- ------- B) Decreased sodium and potassium 125. What nursing assessment of a paralyzed client would indicate the probable presence of a fecal impaction? ----------- Correct Answer ----------- B) Oozing liquid stool 126. A 20 year-old client has an infected leg wound from a motorcycle accident, and the client has returned home from the hospital. The client is to keep the affected leg elevated and is on contact precautions. The client wants to know if visitors can come. The appropriate response from the home health nurse is that: ----------- Correct Answer ------- ---- C) Visitors should wash their hands before and after touching the client 127. A child is admitted to the pediatric unit with a diagnosis of suspected meningococcal meningitis. Which admission orders should the nurse do first? ----------- Correct Answer ----------- Place in respiratory/secretion precautions 128. Which of these nursing diagnoses of 4 elderly clients would place 1 client at the greatest risk for falls? ----------- Correct Answer ----------- Altered patterns of urinary elimination related to nocturia 129. A nurse who is reassigned to the emergency department needs to understand that gastric lavage is a priority in which situation? ----------- Correct Answer ----------- An infant who has been identified to have botulism 130. A newly admitted adult client has a diagnosis of hepatitis A. The charge nurse should reinforce to the staff members that the most significant routine infection control strategy, in addition to hand washing, to be implemented is which of these? ----------- Correct Answer ----------- D) Have gloves on while handling bedpans with feces 131. Which of these clients with associated lab reports is a priority for the nurse to report to the public health department within the next 24 hours? ----------- Correct Answer ----------- An elderly factory worker with a lab report that is positive for acid-fast bacillus smear 132. A client is diagnosed with methicillin resistant staphylococcus aureus pneumonia. What type of isolation is most appropriate for this client? ----------- Correct Answer -------- --- D) Contact 147. Several clients are admitted to an adult medical unit. The nurse would ensure airborne precautions for a client with which medical condition? ----------- Correct Answer ----------- B) A positive purified protein derivative with an abnormal chest x-ray 148. After an unsuccessful resuscitation attempt, the nurse calls the family of the deceased. The family wish to see the body before it is taken to the funeral home. Which interventions should the nurse take to prepare the body before the family enters the room? (Select all that apply) ----------- Correct Answer ----------- C) Place a small pillow under the head D) Remove resuscitation equipment from the room E) Gently close the eyes 149. A client is scheduled to receive an oral solution of radioactive iodine (131I). In order to reduce hazards, the priority information for the nurse to include during the instructions to the client is which of these statements? ----------- Correct Answer ----------- In the initial 48 hours avoid contact with children and pregnant women, and after urination or defecation flush the commode twice. 150. The nurse is assigned to a client newly diagnosed with active tuberculosis. Which of these protocols would be a priority for the nurse to implement? ----------- Correct Answer ----------- Place client in a negative pressure private room and have all who enter the room use masks with shields 151. The charge nurse is planning assignments on a medical unit. Which client should be assigned to the PN? ----------- Correct Answer ----------- C) Irrigate and redress a leg wound 152. When assessing a client, it is important for the nurse to be informed about cultural issues related to the client's background because ----------- Correct Answer ----------- Normal patterns of behavior may be labeled as deviant, immoral, or insane 153. A client who is newly diagnosed with type 2 diabetes mellitus (DM) receives a prescription for metformin (Glucophage) 500 mg PO twice daily. What information should the nurse include in this client's teaching plan? (Select all that apply.) ----------- Correct Answer ----------- B) Recognize signs and symptoms of hypoglycemia. Report persist polyuria to the healthcare provider Take Glucophage with the morning and evening meal. 154. After working with a very demanding client, an unlicensed assistive personnel (UAP) tells the nurse, "I have had it with that client. I just can't do anything that pleases him. I'm not going in there again." The nurse should respond by saying ----------- Correct Answer ----------- C) "He is scared and taking it out on you. Let's talk to figure out what to do." 155. A client with a diagnosis of bipolar disorder has been referred to a local boarding home for consideration for placement. The social worker telephoned the hospital unit for information about the client's mental status and adjustment. The appropriate response of the nurse should be which of these statements? ----------- Correct Answer ----------- D) I need to get the client's written consent before I release any information to you. 156. A client is admitted with a diagnosis of schizophrenia. The client refuses to take medication and states "I don't think I need those medications. They make me too sleepy and drowsy. I insist that you explain their use and side effects." The nurse should understand that ----------- Correct Answer ----------- B) The client has a right to know about the prescribed medications 157. A nurse is administering diazepam, a benzodiazepine, 10 mg IV push PRN, as prescribed to a client with alcohol withdrawal symptoms. Which actions should the nurse implement when administering the medication? (Select all that apply) ----------- Correct Answer ----------- B) Monitor for changes in level of consciousness D) Perform ongoing assessment of respiratory status E) Administer slowly over at least two minutes 158. A client newly diagnosed with diabetes mellitus suddenly becomes confused and weak. Which interventions should the nurse implement? (Select all that apply) ----------- Correct Answer ----------- A) Give the client 4 ounces of orange juice B) Obtain blood pressure and pulse rate E) Check the client's current finger stick blood glucose 159. The nurse is interacting with a female client who is diagnosed with postpartum depression. Which finding should the nurse document as an objective sign of depression? (Select all that apply) ----------- Correct Answer ----------- A. Interacts with a flat affect B. Avoids eye contact C. Has a disheveled appearance 160. A client who is hospitalized and recently is now confused and lethargic. Which actions should the nurse implement? (Select all that apply) ----------- Correct Answer ----- ------ A) Measure capillary glucose level B) Monitor cardiac telemetry pattern E) Initiate fall risk precautions A pt presses the call bell and requests pain medication for a severe headache. To assess the quality of the pt. Pain, which approach should the nurse use? ----------- Correct Answer ------------ Ask the patient to describe the pain The nurse is wearing personal protective equipment while caring for a pt. When exiting the room, which PPE should be removed first? ----------- Correct Answer ------------ Gloves An older pt is brought to the ED with a sudden onset of confusion that occurred after experiencing a fall at home. The daughter, who has power of attorney, has brought the client's prescriptions. Which information should the nurse provide first when reporting to the healthcare provider using SBAR communication? ----------- Correct Answer ------------ Increasing confusion of the pt. A pt tells the nurse about working out with a personal trainer and swimming three times a week in an effort to lose weight and sleep better. The pt states that it still is taking hours to fall asleep at night. Which action should the nurse implement? ----------- Correct Answer ------------ Ask the pt for a description of the exercise schedule that is being followed 2 days prior to discharge from the rehab facility, the nurse is teaching a pt who is recovering from Guillain-Barre syndrome about home. Which actions should the nurse include when providing discharge teaching to the pt and spouse? SATA ----------- Correct Answer ------------ -review safe transfer strategies -develop a nutritional plan -help identify community support The nurse implements a tertiary prevention program for type 2 diabetes in a rural health clinic. Which outcome indicates that the program was effective? ----------- Correct Answer ------------ pt who develop disease complications promptly received rehabilitation The nurse has received funding to design a health promotion project for African American women who are at risk for developing breast cancer. Which resource is most important in designing this program? ----------- Correct Answer ------------ participation of community leaders in planning the program A pt with multiple injuries is being treated in the burn trauma unit just hours after the injuries occurred. The healthcare provider instructs the nurse to avoid auto contamination when performing dressing changes. Which intervention is most important for the nurse to implement? ----------- Correct Answer ------------ use gown, mask, and gloves with dressing changes A pt is recovering in the critical care unit following a cardiac cath. IV nitro and heparin are infusing. The pt is sedated but responds to instructions. After changing positions, the pt complains of pain at the right groin insertion site. What action should the nurse implement? ----------- Correct Answer ------------ Stimulate the pt to take deep breaths Prior to obtaining a trapeze bar for a pt with limited mobility, which pt assessment is most important for the nurse to obtain? ----------- Correct Answer ------------ upper body muscle strength While the nurse is assessing an older pt fall risk, the pt reports living at home alone and never falling. Which action should the nurse take? ----------- Correct Answer ------------ Continue to obtain pt data needed to complete the fall risk survey The nurse has completed the diet teaching of a client who is being discharged following tx of a leg wound. A high protein diet is encouraged to promote wound healing. Which lunch choice by the pt indicates that the teaching was effective? ----------- Correct Answer ------------ A tuna fish sandwich with chips and ice cream When conducting diet teaching for a pt who was dx with a myocardial infarction, which snack foods should the nurse encourage the pt to eat? SATA ----------- Correct Answer - ----------- Fresh Turkey slices and berries chick bouillon soup and toast raw unsalted almonds and apples A pt taking clopidogrel reports the onset of diarrhea. Which nursing action should the nurse implement first? ----------- Correct Answer ------------ Observe the appearance of the stool A pt is dx with Meniere's disease. Which problem should the nurse identify as most important in the plan of care? ----------- Correct Answer ------------ Risk for injury related to vertigo A new nurse preparing to irrigate an IV catheter is attaching a 24 gauge needle. Which action should the charge nurse implement? ----------- Correct Answer ------------ Suggest the nurse use 20g needle A nurse working on an Endocrine Unit should see which client first? ----------- Correct Answer ------------ A pt taking corticosteroids who has become disoriented in the last two hours. After an older pt receives tx for drug toxicity the healthcare provider prescribes a 24 hr creatinine clearance, the nurse notes that the pt serum creatinine is 0.3mg. Which action should the nurse implement? ----------- Correct Answer ------------ Notify the healthcare provider of the results An older adult pt with Systemic inflammatory syndrome (SIRS) has a temp of 101.8, HR of 110, and RR of 24 breaths/min. Which additional finding is most important to report to the healthcare provider? ----------- Correct Answer ------------ Serum Creatinine of 2mg/dl A pt in the third trimester of pregnancy reports that she feels some lumpy places in her breasts and that her nipples sometimes leak a yellowish fluid. She has an appointment with her healthcare provider in two weeks. What actions should the nurse take? ---------- - Correct Answer ------------ Explain that this is normal recreation and can be assessed at the next visit. A pt presents to the labor and delivery unit with a report of leaking fluid that is greenish- brown vaginal discharge. Which action should the nurse take first? ----------- Correct Answer ------------ begin continuous fetal monitoring The mother of a 2 day old infant girl expresses concern about a flea bite type rash on her daughter's body. The nurse identifies a pink papular rash with vesicles superimposed over the thorax, back, buttocks, and abdomen. Which explanation should the nurse offer? ----------- Correct Answer ------------ This is a common newborn rash that will resolve after several days. When assessing a recently delivered mom that her vaginal bleeding is more than expected. Which factor in this history is related to this finding? ----------- Correct Answer ------------ She is a gravid 6 para 5. The nurse is caring for a preterm newborn with nasal flaring, grunting, and sternal retractions. After administering surfactant, which assessment is most important for the nurse to monitor? ----------- Correct Answer ------------ Arterial blood gas A pt who gave birth 48 hours ago has decided to bottle feed teh infant. During the assessment, the nurse observes that both breasts are swollen, warm and tender on palpation. Which instruction should the nurse provide? ----------- Correct Answer ---------- -- Apply ice to the breast for comfort. The nurse is providing discharge teaching to the parents of a 13 month old child who underwent repair for an atrial septal defect. The healthcare provider prescribes aspirin and an antibiotic for the first 6 months postoperatively to prevent infective endocarditis. What information is most important for the nurse discuss with the parents about the child's recovery and prevention of IE. ----------- Correct Answer ------------ Brush the child's teeth everyday and ensure the child receives regular dental follow up. A pt who is admitted with complications related to hypopituitarism is diaphoretic and hypotensive. Which assessment finding warrants immediate intervention by the nurse? - ---------- Correct Answer ------------ Lethargy The nurse identifies an electrolyte imbalance, a weight gain of 4.4lbs in 24 hours and an elevated central venous pressure for a pt with full thickness burns. Which intervention should the nurse implement? ----------- Correct Answer ------------ Auscultate for irregular heart rate. A nurse who is working in the emergency department triage area is presented with four clients at the same time. The client presenting with which symptoms require the most immediate intervention by the nurse? ----------- Correct Answer ------------ Chest discomfort one hour after consuming a large spicy meal. Which self care measure is most important to include in the plan of care of a pt recently diagnosed with type 2 diabetes mellitus? ----------- Correct Answer ------------ blood glucose monitoring. In assessing a pt with type 1 diabetes mellitus, the nurse notes that the client's respirations have changed from 16 breaths/min with a normal depth to 32 breaths/min and deep. And the pt has become lethargic. Which assessment data should the nurse obtain next? ----------- Correct Answer ------------ blood glucose An adolescent who was diagnosed with diabetes mellitus Type 1 at the age of 9 is admitted to the hospital in diabetic ketoacidosis. Which occurrence is the most likely cause of ketoacidosis? ----------- Correct Answer ------------ had a cold and ear infection for the past two days. The nurse assesses a pt who had bilateral total knee replacements four hours ago. The nurse notes that the dressing on the right knee is saturated with serosanguineous drainage. What action should the nurse implement? ----------- Correct Answer ------------ Determine if the wound drainage device is functioning correctly. A pt with chronic kidney disease has an arteriovenous (AV) fistula in the left forearm. Which observation by the nurse indicates that the fistula is patient? ----------- Correct Answer ------------ Auscultation of a thrill on the left forearm A client with urge incontinence was treated with onabotulinumtoxinA injections and is now experiencing urinary retention. Which action should the nurse include in the client's plan of care? ----------- Correct Answer ------------ Remind the pt to practice pelvic floor exercises regularly. A pt is admitted with a dx of urolithiasis. Which finding is most important for the nurse to report to the healthcare provider? ----------- Correct Answer ------------ Hematuria that is beginning to turn pink The nurse is evaluating the chest drainage system of a patient with a chest tube inserted to treat a left hemothorax. Which finding requires intervention by the nurse? ---- ------- Correct Answer ------------ cont. bubbling in the water seal chamber The nurse is caring for a pt who has chronic obstructive pulmonary disease and chest pain related to a recent fall. What nursing intervention requires the greatest caution when caring for a pt with COPD? ----------- Correct Answer ------------ administering narcotics for pain relief The nurse instructs a pt in use of an incentive spirometer. The client performs a return demonstration as seen in the video. Which action should the nurse take in response to the return demonstration? ----------- Correct Answer ------------ Remind the pt to cough using the spirometer The healthcare provider prescribes a sepsis protocol for a pt with organ failure caused by a ruptured appendix. Which intervention is most important for the nurse to include in the plan of care? ----------- Correct Answer ------------ Maintain strict intake and output A pt diagnosed with calcium kidney stones has a history of gout. A new prescription for aluminum hydroxide is scheduled to begin at 0730. Which client medication should the The nurse is developing a plan of care for a pt who reports tingling of the feet and who is newly diagnosed with peripheral vascular disease. Which outcome should the nurse include in the plan of care for this client? ----------- Correct Answer ------------ The pt blood pressure readings will be less than 160/90 Prior to surgery, written consent must be obtained. Which is the nurse's legal responsibility with regard to obtaining written consent? ----------- Correct Answer ---------- -- Determine that the surgical consent form has been signed and is included in the pt record A client presents to the emergency dept with muscle aches, headache, fever, and describes a recent loss of taste and smell. The nurse obtains a swab for COVID -19 testing. Which action is most important for the nurse to take? ----------- Correct Answer -- ---------- place the nasal swab specimen for COVID 19 directly in a biohazard bag. What might the nurse suggest to a pt with fibrocystic breasts in an attempt to help relieve her symptoms? ----------- Correct Answer ------------ increase high calcium foods in diet The nurse is assessing a pt who reported falling 2 days ago and has a history of gouty arthritis that is controlled with allopurinol. The pt states the left knee is swollen and extremely painful to the touch. Which instructions should the nurse include in the discharge teaching? ----------- Correct Answer ------------ decrease consumption of red meat and most seafood. The nurse is teaching a group of women about osteoporosis and exercise. The nurse should emphasize the need for which type of regular activity? ----------- Correct Answer - ----------- Weight bearing exercise A male client with cirrhosis has jaundice and pruritus. He tells the nurse that he has been soaking in hot baths at night with no relief of his discomfort. Which action should the nurse take? ----------- Correct Answer ------------ Encourage the pt to use cooler water and apply calamine lotion after soaking. A pt with rheumatoid arthritis starts a new prescription for subcutaneously once weekly. The nurse emphasizes the importance of reporting which problem to the healthcare provider? ----------- Correct Answer ------------ Joint stiffness An adult female tells the nurse that though she is afraid that her abusive boyfriend might one day kill her, she keeps hoping that he will change. Which action should the nurse take first? ----------- Correct Answer ------------ Explore the pt readiness to discuss the situation A male client approaches the nurse with an angry expression on his face and raises his voice, saying, "My roommate is the most selfish, self-centered, angry person I have ever met If he loses his temper one more time with me, I am going to punch him." The nurse recognizes that the pt is using which defense mechanism? ----------- Correct Answer ----- ------- Projection A young adult male who is being seen at the employee health care clinic for an annual assessment tells the nurse that his mother was diagnosed with schizophrenia when she was his age and that life with a schizophrenic mother was difficult indeed. Which response is best for the nurse to provide? ----------- Correct Answer ------------ Ask the pt if he is worried about becoming schizophrenic at the same age his mother was dx. On admission to the Emergency Department, a female client who was diagnosed with bipolar disorder 3 years ago reports that this morning she took a handful of medications and left a suicide note for her family. Which information is most important for the nurse to obtain? ----------- Correct Answer ------------ When the pt last took drugs for bipolar disorder The nurse is providing care for a pt with schizophrenia who receives haloperidol decanoate 75 mg intramuscularly every 4 weeks. The pt begins developing puckering and smacking of the lips and facial grimacing. Which intervention should the nurse implement? ----------- Correct Answer ------------ Complete abnormal involuntary movement scale A pt with postpartum depression, who is admitted to the behavioral health unit refuses to leave her room or eat meals. In addition to maintaining physical safety, which short term goal should the nurse include in the plan of care? ----------- Correct Answer ---------- -- Attends one group activity per day The psychiatric nurse is caring for pts on an adolescent unit. Which client requires the nurse's immediate attention ----------- Correct Answer ------------ An 18 year old pt with antisocial behavior who is being yelled at by other pts. The nurse is providing education to a client who experiences recurrent levels of moderate anxiety to situations and perceived stress. In addition to information about prescribed medication and administration, which instruction should the nurse include in the teaching? ----------- Correct Answer ------------ Practice using muscle relaxation techniques A preschool aged boy is admitted to the pediatric unit following successful resuscitation from a near drowning incident. While providing care to the child, the nurse begins with the brother who rescued the child from the swimming pool and initiated resuscitation. The nurse notices the older boy becomes withdrawn when asked about what happened. Which action should the nurse take? ----------- Correct Answer ------------ Ask the older brother how he felt during the incident. The nurse is completing the admission assessment of a 3-year old who is admitted with bacterial meningitis and hydrocephalus. Which assessment finding is evidence that the child is experiencing increased intracranial pressure (ICP)? A. Tachycardia and tachypnea B. Sluggish and unequal pupillary responses C. Increased head circumference and bulging fontanels D. Blood pressure fluctuations and syncope ----------- Correct Answer ------------ B. Sluggish and unequal pupillary responses A client with acute pancreatitis is admitted with severe, piercing abdominal pain and an elevated serum amylase. Which additional information is the client most likely to report to the nurse? A. Abdominal pain decreases when lying supine B. Pain lasts an hour and leaves the abdomen tender C. Right upper quadrant pain refers to right scapula D. Drinks alcohol until intoxicated at least twice weekly. ----------- Correct Answer --------- --- A. Abdominal pain decreases when lying supine A child newly diagnosed with sickle cell anemia (SCA) is being discharged from the hospital. Which information is most important for the nurse to provide the parents prior to discharge? A. Instructions about how much fluid the child should drink daily. B. Signs of addiction to opioid pain medications C. Information about non-pharmaceutical pain relief measures D. Referral for social services for the child and family ----------- Correct Answer ------------ A. Instructions about how much fluid the child should drink daily To auscultate for a carotid bruit, the nurse places the stethoscope at what location. (Select the location on the image with a red dot). ----------- Correct Answer ------------ I placed the red dot on the base of the neck on the right side After receiving report on an inpatient acute care unit, which client should the nurse assess first? A. The client with an obstruction of the large intestine who is experiencing abdominal distention B. The client who had surgery yesterday and is experiencing a paralytic ileus with absent bowel sounds C. The client with a small bowel obstruction who has a nasogastric tube that is draining greenish fluid D. The client with a bowel obstruction due to a volvulus who is experiencing abdominal rigidity ----------- Correct Answer ------------ D. The client with a bowel obstruction due to a volvulus who is experiencing abdominal rigidity A teenager presents to the emergency department with palpitations after vaping at a party. The client is anxious, fearful, and hyperventilating. The nurse anticipates the client developing which acid base imbalance? A. Respiratory acidosis B. Metabolic alkalosis C. Metabolic acidosis A client who gave birth 48 hours ago has decided to bottle feed the infant. During the assessment, the nurse observes that both breasts are swollen, warm, and tender on palpation. Which instruction should the nurse provide? A. Apply ice to the breasts for comfort B. Wear a loose-fitting bra during the day to prevent nipple irritation C. Run warm water over breasts D. Express small amounts of milk from the breasts to relieve pressure ----------- Correct Answer ------------ A. Apply ice to the breasts for comfort The nurse is preparing a client who had a below-the-knee (BKA) amputation for discharge to home. Which recommendations should the nurse provide this client? (Select all that apply) A. Avoid range of motion exercises B. Use a residual limb shrinker C. Apply alcohol to the stump after bathing D. Inspect skin for redness E. Wash the stump with soap and water ----------- Correct Answer ------------ B. Use a residual limb shrinker D. Inspect skin for redness E. Wash the stump with soap and water A toddler presenting with a history of intermittent skin rashes, hives, abdominal pain, and vomiting that occurs after ingesting of milk products arrives to the clinic accompanied by the parents. Which type of testing should the nurse provide education to the toddler's family about? A. Serum immunoglobulin E (IgE) B. Intradermal test C. Atopy patch test D. Placebo-controlled food challenge ----------- Correct Answer ------------ A. Serum immunoglobulin E (IgE) A client who is scheduled for a bronchoscopy in the morning is anxious and asking the nurse numerous questions about the procedure. In preparing the client for the procedure, which intervention has the highest priority? A. Allow client to gargle with warm salt water B. Administer a sedative to alleviate anxiety C. Instruct client to write down the questions D. Deny client's request for a midnight snack ----------- Correct Answer ------------ C. Instruct client to write down the questions The nurse assesses a client one hour after starting a transfusion of packed red blood cells and determines that there are no indications of a transfusion reaction. What instruction should the nurse provide the unlicensed assistive personnel (UAP) who is working with the nurse? A. Notify the nurse when the transfusion has finished, so further client assessment can be done B. Continue to measure the client's vital signs every thirty minutes until the transfusion is complete C. Monitor the client carefully for the next three hours and report the onset of a reaction immediately D. Since a reaction did not occur, the priority is to maintain client comfort during the transfusion ----------- Correct Answer ------------ B. Continue to measure the client's vital signs every thirty minutes until the transfusion is complete The healthcare provider prescribes a sepsis protocol for a client with multi-organ failure caused by a ruptured appendix. Which intervention is most important for the nurse to include in the plan of care? A. Assess warmth of extremities B. Keep head of bed raised 45 degrees C. Monitor blood glucose level D. Maintain strict intake and output ----------- Correct Answer ------------ D. Maintain strict intake and output A client presses the call bell and requests pain medication for a severe headache. To assess the quality of the client's pain, which approach should the nurse use? A. Ask the client to describe the pain B. Observe body language and movement C. Identify effective pain relief measures D. Provide a numeric pain scale ----------- Correct Answer ------------ A. Ask the client to describe the pain A client presents to the labor and delivery unit with a report of leaking fluid that is greenish-brown vaginal discharge. Which action should the nurse take first? A. Start an intravenous infusion B. Administer oxygen via facemask C. Perform a vaginal exam D. Begin continuous fetal monitoring ----------- Correct Answer ------------ D. Begin continuous fetal monitoring A client asks the nurse for information about how to reduce risk factors for benign prostatic hyperplasia (BPH). Which information should the nurse provide? A. Consume a high protein diet B. Increase physical activity C. Take vitamin supplements D. Obtain a prostate-specific antigen blood level test ----------- Correct Answer ------------ B. Increase physical activity The healthcare provider prescribes a fluid challenge of 0.9% sodium chloride 1,000 mL to be infused intravenously over 4 hours. The IV administration set delivers 10gtt/mL. How many gtt/minute should the nurse regulate the infusion? (Round to the nearest whole number) ----------- Correct Answer ------------ 42 gtt/min Following a cardiac catheterization and placement of a stent in the right coronary artery, the nurse administers prasugrel, a platelet inhibitor, to the client. To monitor for adverse effects from the medication, which assessment is most important for the nurse to include in this client's plan of care? A. observe color of urine B. Measure body temperature C. Assess skin turgor D. Check for pedal edema ----------- Correct Answer ------------ A. Observe color of urine A client fell in the bathroom when left unattended by the unlicensed assistive personnel (UAP). Which information should the nurse include in the client's health record? A. The UAP left the client to assist another client B. The last time client was assisted to the bathroom C. The unit was understaffed when the client fell D. The client fell sustaining a fracture to the left hip ----------- Correct Answer ------------ D. The client fell sustaining a fracture to the left hip The nurse is reviewing the diagnostic tests prescribed for a client with a positive skin test. Which subjective findings reported by the client supports the diagnosis of tuberculosis? A. Barking cough and vomiting B. Mucopurulent cough and night sweats C. Dry cough and chest tightness D. Chronic cough and fatty stools ----------- Correct Answer ------------ B. Mucopurulent cough and night sweats In assessing a client with type 1 diabetes mellitus, the nurse notes that the client's respirations have changed from 16 breaths/min with a normal depth to 32 breaths/min and deep, and the client become lethargic. Which assessment data should the nurse obtain next? A. Temperature B. Breath sounds C. Blood glucose D. White blood cell count ----------- Correct Answer ------------ C. Blood glucose A nurse receives report on a client who is four hours post-total abdominal hysterectomy. The previous nurse reports that it was necessary to change the client's perineal pad hourly and that it is again saturated. The previous nurse also reports that the client's urinary output has decreased. Which action should the nurse implement first? A. Evaluate the skin turgor B. Assess for weakness or dizziness C. Change the perineal pad D. Measure the urinary output ----------- Correct Answer ------------ B. Assess for weakness or dizziness B. Review with the client proper foot care and prevention of injury C. Teach subcutaneous injection technique, site rotation, and insulin management D. Coordinate carbohydrate controlled meals at consistent times and intervals. E. Mix bedtime dose of insulin glargine with insulin aspart sliding scale dose F. Fingerstick glucose assessments every 6h with meals ----------- Correct Answer ------- ----- B. Review with client proper foot care and prevention of injury C. Teach subcutaneous injection technique, site rotation, and insulin management D. Coordinate carbohydrate controlled meals at consistent times and intervals F. Fingerstick glucose assessments every 6h with meals The psychiatric nurse is caring for clients on an adolescent unit. Which client requires the nurse's immediate attention? A. A 14yo client with anorexia nervosa who is refusing to eat the evening snack B. A 16yo client diagnosed with major depression who refuses to participate in group C. A 17yo client diagnosed with bipolar disorder who is pacing around the lobby D. An 18yo client with antisocial behavior who is being yelled at by other clients ----------- Correct Answer ------------ D. An 18yo client with antisocial behavior who is being yelled at by other clients A client at 12 weeks gestation is admitted to the antepartum unit with a diagnosis of hyperemesis gravidarum. Which action is most important for the nurse to implement? A. Obtain the client's 24-hour dietary recall B. Document mucosal membrane status C. Schedule a consult with a nutritionist D. Initiate prescribed intravenous fluids ----------- Correct Answer ------------ D. Initiate prescribed intravenous fluids A pediatric client is taking the beta-adrenergic blocking agent propranolol. In developing a teaching plan, the nurse should teach the parents to report which sign of overdose? A. Bradycardia B. Tachypnea C. Hypertension D. Coughing ----------- Correct Answer ------------ A. Bradycardia Prior to obtaining a trapeze bar for a client with limited mobility, which client assessment is most important for the nurse to obtain? A. Upper body muscle strength B. Balance and posture C. Risk for disuse syndrome D. Pressure sore risk ----------- Correct Answer ------------ A. Upper body muscle strength A 3-year-old boy was successfully toilet trained prior to his admission to the hospital for injuries sustained from a fall. His parents are very concerned that the child has regressed in his toileting behaviors. Which information should the nurse provide to the parents? A. A retraining program will need to be initiated when the child returns home. B. Diapering will be provided since hospitalization is stressful to preschoolers C. A potty chair should be brought from home so he can maintain his toileting skills D. Children usually resume their toileting behaviors when they leave the hospital --------- -- Correct Answer ------------ D. Children usually resume their toileting behaviors when they leave the hospital The nurse is managing the care of a client with Cushing's syndrome. Which interventions should the nurse delegate to the unlicensed assistive personnel (UAP)? (Select all that apply) A. Report any client complaint of pain or discomfort B. Evaluate the client for sleep disturbances C. Assess the client for weakness and fatigue D. Weigh the client and report any weight gain E. Note and report the client's food and liquid intake during meals and snacks ----------- Correct Answer ------------ A. Report any client complaint of pain or discomfort D. Weigh the client and report any weight gain E. Note and report the client's food and liquid intake during meals and snacks A young adult visits the client reporting symptoms associated with gastritis. Which information in the client's history is most important for the nurse to address in the teaching plan? A. Consumes 10 or more drinks of alcohol every weekend B. Snacks on foods with very high salt content on a daily basis C. Exercises vigorously every evening right before going to bed D. Recently became a vegetarian and eats a lot of high fiber foods ----------- Correct Answer ------------ A. Consumes 10 or more drinks of alcohol every weekend After administering a proton pump inhibitor (PPI), which action should the nurse take to evaluate the effectiveness of the medication? A. Auscultate for bowel sounds in all quadrants B. Ask the client about gastrointestinal pain C. Monitor the client's serum electrolyte levels D. Measure the client's fluid intake and output ----------- Correct Answer ------------ B. Ask the client about gastrointestinal pain When assessing a recently delivered, multigravida client, the nurse finds that her vaginal bleeding is more than expected. Which factor in this client's history is related to this finding? A. The second stage of labor lasted 10 minutes B. She received butorphanol 2mg IVP during labor C. She is over 35 years of age D. She is a gravida 6, para 5 ----------- Correct Answer ------------ D. She is a gravida 6, para 5 When assessing an IV site that is used for fluid replacement and medication administration, the client complains of tenderness when the arm is touched above the site. Which additional assessment finding warrants immediate intervention by the nurse? A. Client uses the arm cautiously B. Red streak tracking the vein C. A sluggish blood return D. Spot of dried blood at insertion site ----------- Correct Answer ------------ B. Red streaks tracking the vein An older adult male reporting abdominal pain is admitted to the hospital from a long- term care facility. It has been 7 days since his last bowel movement, his abdomen is distended, and he just vomited 150mL of dark brown emesis. In what order should the nurse implement these interventions? (Highest to lowest priority) ----------- Correct Answer ------------ 1. Send emesis sample to the lab 2. Elevate the head of the bed 3. Complete focused assessment 4. Offer PRN pain medication When taking a health history, which information collected by the nurse correlates most directly to a diagnosis of chronic peripheral arterial insufficiency? A. History of intermittent claudication B. A positive Brodie-Trendelenburg test C. Ankle ulceration and edema D. A serum cholesterol level of 250mg/dl (6.47mmol/L) ----------- Correct Answer ---------- -- A. History of intermittent claudication The nurse is providing discharge teaching to the parents of a 13 month old child who underwent repair for an atrial septal defect. The healthcare provider prescribes aspirin and an antibiotic for the first 6 months postoperatively to prevent infective endocarditis (IE). What information is most important for the nurse discuss with the parents about the child's recovery and prevention of IE? A. Refer the mother to the healthcare provider to discuss infective endocarditis B. Brush the child's teeth every day and ensure the child receives regular dental followup C. Give the child acetaminophen for pain or fever and visit the surgeon for follow-up D. Monitor the child for regular bowel movements and urine output that exceeds intake - ---------- Correct Answer ------------ B. Brush the child's teeth every day and ensure the child receives regular dental followup An unlicensed assistive personnel (UAP) is assigned to ambulate a client with influenza who has droplet precautions implemented. The UAP requests a change in assignment, stating the reason of having not been fitted yet for a N95 respirator mask. Which action should the nurse take? A. send the UAP to be fitted for a particulate filter mask immediately so she can provide care to this client. B. Instruct the UAP that a standard face mask is sufficient for the provision of care for the assigned client C. Ask the client for a description of the exercise schedule that is being followed D. Determine the amount of weight the client has lost since increasing activity ----------- Correct Answer ------------ C. Ask the client for a description of the exercise schedule that is being followed The nurse is developing an educational program for older clients who are being discharged with new antihypertensive medications. The nurse should ensure that the education materials include which characteristics? (Select all that apply) A. Uses common words with few syllables B. Printed using a 12-point type font C. Uses pictures to help illustrate complex ideas D. Contains a list with definitions of unfamiliar terms E. Written at a twelfth-grade reading level ----------- Correct Answer ------------ A. Uses common words with few syllables C. Uses pictures to help illustrate complex ideas D. Contains a list with definitions of unfamiliar terms The nurse is providing care for a client with severe peripheral arterial disease (PAD). The client reports a history of rest ischemia, with leg pain that occurs during the night. Which action should the nurse take in response to this finding? A. Elevate the legs to assess for color changes B. Provide a heating pad for PRN use C. Offer cold packs when the pain occurs D. Suggest dangling the legs when pain occurs ----------- Correct Answer ------------ C. Offer cold packs when the pain occurs The nurse assess a client being treated for Herpes zoster (shingles). Which assessments should the nurse include when evaluating the effectiveness of the the treatment? (Select all that apply) A. Functional ability B. Skin integrity C. Pain scale D. Bowel sounds E. heart sounds ----------- Correct Answer ------------ A. Functional ability B. Skin integrity A heparin infusion is prescribed for a client who weighs 220 pounds. After administering a bolus dose of 80 units/kg, the nurse calculates the infusion rate for the heparin solution at 18 units/kg/hr. The available solution is Heparin Sodium 25,000 units in 5% Dextrose injection 250mL. The nurse should program the infusion pump to deliver how many mL/hour? ----------- Correct Answer ------------ 18 When providing client care the nurse identifies a problem and develops a related clinical question. Next, the nurse intends to gather evidence so that the decision-making process in response to the problem and clinical question is evidence-based. When gathering evidence, which consideration is most important? A. Past experience with similar problems B. Relevance to the situation C. Related personal values D. Frequency that the problem occurs ----------- Correct Answer ------------ B. Relevance to the situation A client diagnosed with calcium kidney stones has a history of gout. A new prescription for aluminum hydroxide is scheduled to begin at 0730. Which client medication should the nurse bring to the healthcare provider's attention? A. Esinapril B. Allopurinol C. Furosemide D. Aspirin, low dose ----------- Correct Answer ------------ B. Allopurinol A client with urge incontinence was treated with onabotuilinumtoxinA injections and is now experiencing urinary retention. Which action should the nurse include in the client's plan of care? A. Provide a bedside commode for immediate use in the client's room B. Teach the client techniques for performing intermittent catheterization C. Explain the need to limit intake of oral fluids to reduce client discomfort D. Remind the client to practice pelvic floor (Kegel) exercises regularly ----------- Correct Answer ------------ D. Remind the client to practice pelvic floor (Kegel) exercises regularly After a spider bite on the lower extremity, a client is admitted for treatment of an infection that is spreading up the leg. Which admission assessment findings should the nurse report to the healthcare provider? (Select all that apply) A. Location of the initial IV site B. Red blood cell count (RBC) C. Swollen lymph nodes in the groin D. White blood cell count (WBC) E. Core body temperature ----------- Correct Answer ------------ C. Swollen lymph nodes in the groin D. White blood cell count (WBC) E. Core body temperature The home care nurse visits a client who has cancer. The client reports having a good appetite but experiencing nausea when smelling food cooking. Which action should the nurse implement? A. Encourage family members to cook meals outdoors and bring the cooked food inside B. Assess the client's mucous membranes and report the findings to the healthcare provider C. Advise the client to replace cooked foods with a variety of different nutritional supplements D. Instruct the client to take an antiemetic before every meal to prevent excessive vomiting ----------- Correct Answer ------------ A. Encourage family members to cook meals outdoors and bring the cooked food inside The nurse is wearing personal protective equipment (PPE) while caring for a client. When exiting the room, which PPE should be removed first? A. Gloves B. Mask C. Eyewear D. Gown ----------- Correct Answer ------------ A. Gloves An older male client, who is a retired chef, is hospitalized with a diabetic ulcer on his foot. His daughter tells the nurse that her father has become increasingly obsessed with the way his food is prepared in the hospital. The nurse's response should be based on what information? A. The client probably has an organic brain disease and will likely have Alzheimer's disease within a few years B. The family needs a social worker to talk to them about how to handle their father when he becomes annoying C. The daughter is under stress and should be encouraged to think about happier times D. If the client was compulsive about food when he was younger, the aging process can magnify this ----------- Correct Answer ------------ D. If the client was compulsive about food when he was younger, the aging process can magnify this A client is receiving enoxaparin 30mg subcutaneously twice a day. In assessing for adverse effects of the medication, which serum laboratory value is most important for the nurse to monitor? A. Glucose B. Calcium C. Platelet count D. White blood cell count ----------- Correct Answer ------------ C. Platelet count The nurse is caring for a 24-month-old toddler who has sensory sensitivity, difficulty engaging in social interactions, and has not yet spoken two-word phrases. Which assessment should the nurse administer? A. The modified checklist for autism in toddlers (M-CHAT) B. Psychology Systems Questionnaire (PHQ-2) C. Behavioral Style Questionnaire (BSQ) D. The Ages and Stages Questionnaire (ASQ) ----------- Correct Answer ------------ A. The Modified Checklist for Autism in Toddlers (M-CHAT) Prior to surgery, written consent must be obtained. Which is the nurse's legal responsibility with regard to obtaining written consent? A. Explain the surgical procedure to the client and ask the client to sign the consent form B. Ask the client or a family member to sign the surgical consent form C. Determine that the surgical consent form has been signed and is included in the client's record. D. Note the duration of the seizure E. Restrain the client ----------- Correct Answer ------------ A. Loosen restrictive clothing C. Ease the client to the floor D. Note the duration of the seizure On admission to the Emergency Department, a female client who was diagnosed with bipolar disorder 3 years ago reports that this morning she took a handful of medications and left a suicide note for her family. Which information is most important for the nurse to obtain? A. Which family member has the client's suicide note B. What drugs the client used for the suicide attempt C. When the client last took drugs for bipolar disorder D. Whether the client over attempted suicide in the past ----------- Correct Answer --------- --- C. When the client last took drugs for bipolar disorder The nurse has complete the diet teaching of a client who is being discharged following treatment of a leg wound. A high protein diet is encouraged to promote wound healing. Which lunch choice by the client indicates that the teaching was effective? A. A tuna fish sandwich with chips and ice cream B. A salad with three kinds of lettuce and fruit C. A peanut butter sandwich with soda and cookies D. Vegetable soup, crackers, and milk ----------- Correct Answer ------------ A. A tuna fish sandwich with chips and ice cream The nurse has received funding to design a health promotion project for African- American women who are at risk for developing breast cancer. Which resource is most important in designing this program? A. A listing of African-American women who live in the community B. Morbidity data for breast cancer in women of all races C. Participation of community leaders in planning the program D. Technical assistance to produce a video on breast self-examination ----------- Correct Answer ------------ C. Participation of community leaders in planning the program A new nurse preparing to irrigate an intravenous catheter is attaching a 24-gauge needle. Which action should the charge nurse implement? A. Suggest the nurse use a 20-gauge needle B. Instruct the nurse to remove the needle C. Direct the nurse to change the IV tubing D. Prompt the nurse to apply povidone to the site ----------- Correct Answer ------------ A. Suggest the nurse use a 20-gauge needle After reviewing the Braden Scale findings of residents at a long-term facility, the charge nurse should to tell the unlicensed assistive personnel (UAP) to prioritize skin care for which client? A. A poorly nourished client who requires liquid supplements B. An older adult who is unable to communicate elimination needs C. A woman with osteoporosis who is unable to bear weight D. A older man whose sheets are damp each time he is turned ----------- Correct Answer ------------ D. A older man whose sheets are damp each time he is turned An increased number of elderly persons are electing to undergo a new surgical procedure which cures glaucoma. Which effect is the nurse likely to note as a result of this increase in glaucoma surgeries? A. Decreased prevalence of glaucoma in the population B. Increased incidence of glaucoma in the population C. Decreased morbidity in the elderly population D. Increased mortality in the elderly population ----------- Correct Answer ------------ A. Decreased prevalence of glaucoma in the population An adult female client tells the nurse that though she is afraid her abusive boyfriend might one day kill her, she keeps hoping that he will change. Which action should the nurse take first? A. Explore client's readiness to discuss the situation B. Determine the frequency and type of client's abuse C. Report the finding to the police department D. Discuss treatment options for abusive partners ----------- Correct Answer ------------ A. Explore client's readiness to discuss the situation A client's morning assessment includes bounding peripheral pulses, weight gain of 2 pounds (0.91 kg), pitting ankle edema, and moist crackles bilaterally. Which intervention is most important for the nurse to include in this client's plan of care? A. Maintain accurate intake and output B. Administer prescribed diuretic C. Weigh client every morning D. Restrict daily fluid intake to 1500mL ----------- Correct Answer ------------ B. Administer prescribed diuretic The nurse is evaluating the chest drainage system of a client with a chest tube inserted to treat a left hemothorax. Which finding requires intervention by the nurse? A. Rise and fall of water level with respiration B. Continuous bubbling in the water-seal chamber C. Total fluid level in water-seal chamber unchanged D. An average collection of 50 mL/hr drainage ----------- Correct Answer ------------ B. Continuous bubbling in the water-seal chamber A client in the emergency center demonstrates rapid speech, flight of ideas, and reports sleeping only three hours during the past 48 hours. Based on these findings, it is most important for the nurse to review the laboratory value for which medication? A. Lorazepam B. Fluoxetine C. Divalproex D. Olanzapine ----------- Correct Answer ------------ C. Divalproex A client with bacterial meningitis is receiving phenytoin. Which assessment finding indicates to the nurse that the client is experiencing a therapeutic response? ----------- Correct Answer ------------ B. Normal electroencephalogram after drug administration The nurse is demonstrating correct transfer procedures to the unlicensed assistive personnel (UAP) working on a rehabilitation unit. The UAP asks the nurse how to safely move a physically disabled client from the wheelchair to a bed. Which action should the nurse recommend? A. apply a gait belt around the client's waist once a standing position has been assumed B. Place the client's locked wheelchair on the client's strong side next to the bed C. Pull the client into position by reaching from the opposite side of the bed D. Hold the client at arm's length while transferring to better distribute the body weight -- --------- Correct Answer ------------ B. Place the client's locked wheelchair on the client's strong side next to the bed The nurse is auscultating a client's lung sounds. Which description should the nurse use to document this sound? A. Stridor B. Low pitched or coarse crackles C. High pitched or fine crackles D. High pitched wheeze ----------- Correct Answer ------------ C. High pitched or fine crackles A client taking clopidogrel reports the onset of diarrhea. Which nursing action should the nurse implement first? A. Observe the appearance of the stool B. Assess the elasticity of the client's skin C. Review the client's laboratory values D. Auscultate the client's bowel sounds ----------- Correct Answer ------------ A. Observe the appearance of the stool A female client with a history of heart failure (HF) arrives at the clinic after what she describes as a very long trip. Following the initial physical assessment and chart review, which priority action should the nurse implement? A. Administer the prescribed diuretic B. Give a potassium supplement C. Reteach medication regimen D. Auscultate lung and heart sounds ----------- Correct Answer ------------ A. Administer the prescribed diuretic The nurse is preparing a client for discharge who was hospitalized with an acute flare of systemic lupus erythematosus (SLE) symptoms. Which instruction is most important for the nurse to include? A. Use a walker when weakness occurs B. Take prescribed cortisone accurately C. Review the client's current food and medication allergies D. Ask if the mother is experiencing any pain with urination E. Determine if the mother has recently experienced a fall. ----------- Correct Answer ----- ------- A. Encourage increased intake of high protein foods B. Instruct the daughter to check her mother's temperature D. Ask if the mother is experiencing any pain with urination The nurse who works in labor and delivery is reassigned to the cardiac care unit for the day because of a low census in labor and delivery. Which assignment is best for the charge nurse to give this nurse? A. Assist cardiac nurses with their assignments B. Monitor the central telemetry C. Perform the admission of a new client D. Transfer a client to another unit ----------- Correct Answer ------------ A. Assist cardiac nurses with their assignments A client with Type 1 diabetes mellitus and a large draining ulcer of the right foot is admitted with a suspected Staphylococcus aureus infection. Which interventions should the nurse implement? (Select all that apply) A. Monitor the client's white blood cell count B. Explain the purpose of a low bacteria diet C. Send wound drainage for culture and sensitivity D. Institute contact precautions for staff and visitors E. Use standard precautions and wear a mask ----------- Correct Answer ------------ A. Monitor the client's white blood cell count C. Send wound drainage for culture and sensitivity D. Institute contact precautions for staff and visitors The nurse is managing 4 clients in the intensive care unit who are mechanically ventilated. After performing a quick visual assessment, the nurse should prioritize care for the client who is exhibiting which finding? A. An audible voice when client is trying to communicate B. High pressure alarm sounds when client is coughing C. Restrained and restless with a low volume alarm sounding D. Diminished breath sounds in the right posterior base ----------- Correct Answer --------- --- C. Restrained and restless with a low volume alarm sounding A male client tells the nurse that he is concerned that he may have a stomach ulcer, because he is experiencing heartburn and a dull gnawing pain that is relieved when he eats. Which is the best response by the nurse? A. Instruct the client that these mild symptoms can generally be controlled with changes in his diet B. Advise the client that he needs to seek immediate medical evaluation and treatment of these symptoms C. Encourage the client to obtain a complete physical exam, since these symptoms are consistent with an ulcer D. Assure the client that his symptoms may only reflect reflux, since ulcer pain is not relieved with food ----------- Correct Answer ------------ C. Encourage the client to obtain a complete physical exam, since these symptoms are consistent with an ulcer The nurse is evaluating the diet teaching of a client with hypertension. What dinner selection indicates that the client understands the dietary recommendations for hypertension? A. Grilled steak, baked potato with sour cream, green beans, coffee, and raisin cream pie B. Baked pork chops, applesauce, corn on the cob, 1% milk, and key-lime pie C. Tomato soup, grilled cheese sandwich, pickles, skim milk, and lemon meringue pie D. Beef stir fry, fried rice, egg drop soup, diet soda, and pumpkin pie ----------- Correct Answer ------------ B. Baked pork chops, applesauce, corn on the cob, 1% milk, and key- lime pie A client is admitted with a diagnosis of urolithiasis. Which finding is most important for the nurse to report to the healthcare provider? A. Volume of each voiding is more than 300mL B. Serum potassium that is elevated C. Relief of flank pain that radiated into the groin D. Hematuria that is beginning to turn pink ----------- Correct Answer ------------ D. Hematuria that is beginning to turn pink Three days after initiating parenteral fluids for a newborn with a ventricular septal defect (VSD), the nurse assesses an increase in heart rate and blood pressure. Which intervention is most important for the nurse to implement? A. View the graph of daily weights B. Restrict intake of oral fluids C. Assess bilateral lung sounds D. Decrease IV flow rate ----------- Correct Answer ------------ B. Restrict intake of oral fluids During an admission assessment, a client reports currently using heroin. Which information is most important for the nurse to consider in the plan of care? A. History of suicide attempts B. Feelings of disorientation C. Undiagnosed social anxiety symptoms (SAD) D. Family history of schizophrenia ----------- Correct Answer ------------ A. History of suicide attempts The healthcare provider prescribes penicillin G benzathine 2,400,000 units intramuscularly for a client who has a postoperative wound infection. The prefilled syringe is labeled, penicillin G benzathine 1,200,000 units/2mL. How many mL should the nurse administer to this client? ----------- Correct Answer ------------ 4mL A client who experienced a cerebrovascular accident (CVA) is aphasic and has left sided paralysis. Which nurse should be responsible for coordinating the progression of this client's care? A. Nurse case manager B. Adult nurse practitioner C. Neurology unit supervisor D. Risk management nurse ----------- Correct Answer ------------ B. Adult nurse practitioner A client who is admitted with complications related to hypopituitarism is diaphoretic and hypotensive. Which assessment finding warrants immediate intervention by the nurse? - ---------- Correct Answer ------------ Lethargy A client with postpartum depression, who is admitted to the behavioral health unit, refuses to leave her room or eat meals. In addition to maintaining physical safety, which short-term goal should the nurse include in the plan of care? A. Sleeps at least 6 hours per night B. Consumes 3 meals and 1500 mL of fluid per day C. Engages in one client to client interaction daily D. Attends one group activity per day ----------- Correct Answer ------------ D. Attends one group activity per day A 7-year old is admitted to the hospital with persistent vomiting, and a nasogastric tube attached to low intermittent suction is applied. Which finding is most important for the nurse to report to the healthcare provider? A. Shift intake of 640mL IV fluids plus 30mL PO ice chips B. Serum pH of 7.45 C. Gastric output of 100 mL in the last 8 hours D. Serum potassium of 3.0 mg/dL ----------- Correct Answer ------------ D. Serum potassium of 3.0 mg/dL A male client approaches the nurse with an angry expression on his face and raises his voice, saying "My roommate is the most selfish, self-centered, angry person I have ever met and if he loses his temper one more time with me, I am going to punch him out!" The nurse recognizes that the client is using which defense mechanism? A. Splitting B. Projection C. Rationalization D. Denial ----------- Correct Answer ------------ B. Projection The nurse is teaching the client about home care after surgery for an ileal conduit placement. When reviewing the information, which statement should the nurse recognize as needing additional education? A. report presence of mucus in the urine B. Empty pouch when it is half full C. Look at the stoma when replacing appliance The nurse is arranging home care for an older client who has a new colostomy following a large bowel resection three days ago. The client plans to live with a family member. Which actions should the nurse implement? (Select all that apply) A. Teach care of ostomy to care provider B. Assess the client for self care ability C. Provide pain medication instructions D. Request a home safety inspection E. Call home care agency to set up oxygen ----------- Correct Answer ------------ A. Teach care of ostomy to care provider B. Assess the client for self care ability C. Provide pain medication instructions The nurse is caring for a client with the sexually transmitted infection (STI) chlamydia. The client reports having sex with someone who had many partners. Which response should the nurse provide? A. Inform that follow-up may end after the treatment is finished B. Reassure that complications will not occur if the infection is treated C. Notify that persons with STIs are reported to local health departments D. Explain how the infection is transmitted and the health risks involved ----------- Correct Answer ------------ A. Inform that follow-up may end after the treatment is finished. In evaluating the effectiveness of a postoperative client's intermittent pneumatic compression devices, which assessment is most important for the nurse to complete? A. Monitor the amount of drainage from the client's incision B. Observe both lower extremities for redness and swelling C. Evaluate the client's ability to use an incentive spirometer D. Palpate all peripheral pulse points for volume and strength ----------- Correct Answer - ----------- B. Observe both lower extremities for redness and swelling The nurse is caring for a client who is still experiencing light sedation after undergoing an emergency colectomy for bowel obstruction. Which postoperative pain intervention should the nurse implement first? A. Review medical records to obtain pain tolerance expectations B. Attempt to obtain a self-report of pain level from the client C. Provide the first medication prescribed for pain management D. Wait until the client is awake before providing pain management ----------- Correct Answer ------------ B. Attempt to obtain a self-report of pain level from the client The nurse assessing a client who reports falling 2 days ago and has a history of gouty arthritis that is controlled with allopurinol. The client states the left knee is swollen and extremely pain to touch. Which instruction should the nurse include in the discharge teaching? A. Decrease consumption of red meat and most seafood B. Substitute natural fruit juices for carbonated drinks C. Limit use of mobility equipment to avoid muscle atrophy D. Use electric heating pad when pain is at its worse ----------- Correct Answer ------------ A. Decrease consumption of red meat and most seafood The nurse on a pediatric unit observes a distraught mother in the hallway scolding her 3 year old son for wetting his pants. What initial action should the nurse take? A. Provide disposable training pants while calming the mother B. Refer the mother to a community parent education program C. Inform the mother that toilet training is slower for boys D. Suggest that the mother consult a pediatric nephrologist ----------- Correct Answer ---- -------- C. Inform the mother that toilet training is slower for boys The nurse is caring for a client with heart failure. Which method is used in computing the cardiac index to measure how the client's heart is functioning? A. Mean arterial pressure minus right atrial pressure B. Cardiac output divided by body surface area C. Stroke volume divided by end diastolic volume D. Stroke volume multiplied by heart rate ----------- Correct Answer ------------ B. Cardiac output divided by body surface area An older adult client with systemic inflammatory response syndrome (SIRS) has a temperature of 101.8F, heart rate of 110 beats/minute, and respiratory rate of 24 breaths/minute. Which additional finding is most important to report to the healthcare provider? A. Capillary glucose reading of 110 mg/dL B. Serum creatinine of 2.0 mg/dL C. Blood pressure of 130/88 mmHg D. Hemoglobin of 12 g/dL ----------- Correct Answer ------------ B. Serum creatinine of 2.0 mg/dL The nurse leading a care team on a medical surgical unit is assigning client care to a practical nurse (PN) and an unlicensed assistive personnel (UAP). Which task should the nurse delegate to the UAP? A. Evaluate a client's mobility progress toward the plan of care B. Assess for side effects of administered pain medications C. Turn and reposition a client with a total hip replacement D. Monitor an intravenous infusion rate on an established schedule ----------- Correct Answer ------------ C. Turn and reposition a client with a total hip replacement After an older client receives treatment for drug toxicity, the healthcare provider prescribes a 24-hour creatinine clearance test. Prior to starting the urine collection, the nurse notes that the client's serum creatinine is 0.3 mg/dL. Which action should the nurse implement? A. Evaluate the client's serum BUN level B. Initiate the urine collection as prescribed C. Notify the healthcare provider of the results D. Assess the client for signs of hypokalemia ----------- Correct Answer ------------ C. Notify the healthcare provider of the results The nurse is developing a plan of care for a client who reports tingling of the feet and who is newly diagnosed with peripheral vascular disease. Which outcome should the nurse include in the plan of care for this client? A. The client's skin on the lower legs will be intact at the next clinic visit B. The client will express acceptance of their newly diagnosed health status C. The client's blood pressure readings will be less than 160/90 mmHg D. The nurse will encourage the client to walk thirty minutes every day ----------- Correct Answer ------------ C. The client's blood pressure readings will be less than 160/90 mmHg The nurse is preparing a discharge teaching plan for a client who had a liver transplant. Which instruction is most important to include in this plan? A. Increase activity and exercise gradually, as tolerated B. Limit intake of fatty foods for one month after surgery C. Avoid crowds for first two months after surgery D. Notify the healthcare provider if edema occurs ----------- Correct Answer ------------ C. Avoid crowds for first two months after surgery What might the nurse suggest to a client with fibrocystic breasts in an attempt to help relieve her symptoms? A. "Eliminate caffeine from your diet" B. "Avoid vigorous physical exercise immediately after your menstrual periods" C. "Eat a low-carbohydrate, high-protein diet" D. "Increase high-calcium foods in your diet" ----------- Correct Answer ------------ D. "Increase high-calcium foods in your diet" When conducting diet teaching for a client who is on a postoperative full liquid diet, which foods should the nurse encourage the client to eat? (Select all that apply) A. Cheese B. Tea C. Lentils D. Whole grain breads E. Potato soup ----------- Correct Answer ------------ B. Tea C. Lentils E. Potato soup The mother of a 2 day old infant girl expresses concern about a "flea bite" type rash on her daughter's body. The nurse identifies a pink papular rash with vesicles superimposed over the thorax, back, buttocks, and abdomen. Which explanation should the nurse offer? A. This is a common newborn rash that will resolve after several days B. The rash is due to distended oil glands that will resolve in a few weeks C. The healthcare provider is being notified about the rash